Download as pdf or txt
Download as pdf or txt
You are on page 1of 51

TEST -6 (3183)

VISION IAS
PT TEST -6
(3183)
Modern India

2021
SYLLABUS

▪ India in the late Eighteenth Century


▪ The Advent of the Europeans and British Conquest of India
▪ The British Administrative Structure, Reform, Impact and
Organization of Government of India
▪ Social – Religious Movements in 19th – 20th centuries
Freedom Struggle
▪ The Revolt of 1857 & change after 1857
▪ Moderate Phase (1885 – 1905)
Extremist Phase (1905 – 17)
▪ Emergence of Extremist & Leaders
▪ The Partition of Bengal
▪ Morley-Minto Reforms
▪ Swadeshi Movement, Home Rule Movement etc.
The Gandhian Era (1917– 47)
▪ Gandhi – Advent and Achievement
▪ Rowlatt Act, Jallianawala Bagh Massacre and Khilafat Movement
▪ Non- Cooperation, Civil Disobedience and Quit India Movement
▪ British Response and Negotiations
▪ INA and Subhash Chandra Bose
Miscellaneous & Other Dimensions
▪ Reforms / Acts / Committee, Congress Session, Governor –General
Viceroys, Policies, Tribal & Peasant Movements etc.
+ Current Affairs (July 2020)

RESOURCES

▪ Old NCERT: Modern India by Bipin Chandra


▪ New NCERT – Class XIIth Themes in Indian History – Part III
▪ India’s struggle for Independence – Bipin Chandra
TEST 6
https://t.me/UPSC_PDF Download From > https://upscpdf.com https://t.me/UPSC_PDF

1. Consider the following statements about the 4. The Chiang Mai Initiative, sometimes seen
Indian National Army (INA): in the news, is a
1. The idea of INA was first conceived (a) financial swap mechanism among
by Mohan Singh, an officer of the
ASEAN countries.
British Indian army.
(b) an initiative to develop partnerships for
2. Recruits were sought only from Indian
prisoners of war and ex-army men in the Belt and Road Initiative by China.
British forces. (c) regional initiative to promote tourism in
3. INA was headquartered in Calcutta. the East and Southeast Asian region.
Which of the statements given above is/are (d) an annual naval exercise of Pacific rim
correct?
countries led by China.
(a) 1 and 2 only
(b) 1 only
5. Consider the following statements about the
(c) 2 and 3 only
(d) 1, 2 and 3 Cripps Mission proposals, 1941:
1. It outrightly rejected the demand for a
2. Consider the following statements about separate Pakistan put forward by Muslim
Congress Socialist Party: League.
1. It was founded by the efforts of 2. It proposed that a constitution-making
Jayaprakash Narayan, Minoo Masaani
body after the second world war would
and others.
be set up.
2. It worked within the Congress Party and
accepted Congress as the primary body Which of the statements given above is/are
leading the National Movement. correct?
Which of the statements given above is/are (a) 1 only
correct? (b) 2 only
(a) 1 only (c) Both 1 and 2
(b) 2 only
(d) Neither 1 nor 2
(c) Both 1 and 2
(d) Neither 1 nor 2
6. Consider the following statements about

3. Which of the following was/were the 1937 elections:


provisions of the Government of India Act, 1. Only provincial elections were held and
1935? no elections were held at the federal
1. It proposed an All India Federation. level.
2. It provided for the abolition of dyarchy
2. Congress Ministries were formed in all
at the provincial level.
the provinces except Bengal and
3. It provided residuary powers to the
Punjab.
Viceroy.
Select the correct answer using the code Which of the statements given above is/are
given below. correct?
(a) 1 and 2 only (a) 1 only
(b) 2 and 3 only (b) 2 only
(c) 1 and 3 only
(c) Both 1 and 2
(d) 1, 2 and 3
(d) Neither 1 nor 2
2 www.visionias.in ©Vision IAS

Google it:- https://upscpdf.com


https://t.me/UPSC_PDF Download From > https://upscpdf.com https://t.me/UPSC_PDF

7. With reference to Genetically Modified 10. Who among the following was called
Crops (GM crops) in India, often seen in the ‘Lokahitavadi’?
news, consider the following statements: (a) Keshub Chandra Sen
1. They are regulated in India by the (b) Mahatma Phule
National Biodiversity Authority (NBA). (c) Ishwar Vidyasagar
2. Bt cotton is the only GM crop allowed (d) Gopal Hari Deshmukh
for commercial cultivation in India.
Which of the statements given above is/are 11. Consider the following tribal movements:
correct? 1. Santhal rebellion under Sidhu and
(a) 1 only Kanhu
(b) 2 only 2. Munda rebellion under Birsa Munda
(c) Both 1 and 2 3. Uprising by Kols of Chhotanagpur
(d) Neither 1 nor 2 Which of the following is the correct
chronological sequence of the above-given
8. Consider the following statements with movements?
regard to the development of means of (a) 1-2-3
communication in the 19th century: (b) 2-3-1
1. The railways were primarily laid to (c) 1-3-2
promote the linkage of backward regions (d) 3-1-2
with the presidencies.
2. The postage stamps in India were 12. In the context of revolutionary activity
introduced by Lord Dalhousie. during the freedom struggle, consider the
3. The first telegraph line in India, which following pairs:
was an experimental line, was Freedom fighter Conspiracy case
established between Calcutta and 1. Bhagat Singh : Lahore
Diamond Harbour. Conspiracy case
Which of the statements given above are 2. Ramprasad Bismil : Kakori
correct? Conspiracy case
(a) 1 and 2 only 3. Rajguru : Kanpur Bolshevik
(b) 2 and 3 only Conspiracy case
(c) 1, 2 and 3 Which of the pairs given above is/are
(d) 1 and 3 only correctly matched?
(a) 1 only
(b) 1 and 2 only
9. Consider the following statements with
(c) 2 and 3 only
reference to All India States’ People’s
(d) 1, 2 and 3
Conference (AISPC):
1. It's objective was to coordinate political
13. What was the purpose behind issuing Delhi
activities and to raise demands for
Manifesto in 1929?
constitutional changes in the Princely
(a) To organise military coups against the
States.
British empire through military revolt.
2. It was headquartered at Lucknow.
(b) To safeguard the political rights of
Which of the statements given above is/are
Muslims in British India.
correct?
(c) To sever ties with Britain and demand
(a) 1 only
complete independence for India.
(b) 2 only
(d) To demand implementation of
(c) Both 1 and 2
'Dominion Status' in the Round Table
(d) Neither 1 nor 2
Conferences.
3 www.visionias.in ©Vision IAS

Google it:- https://upscpdf.com


https://t.me/UPSC_PDF Download From > https://upscpdf.com https://t.me/UPSC_PDF

14. Consider the following pairs: 18. Who among the following was the founder
Newspaper Associated personality of the ‘Young Bengal Movement in the
1. Amrita Bazar : Sisir Kumar Ghosh 1820’s?
Patrika (a) David Hare
2. Sudharak : Dadabhai Naoroji (b) Keshub Chandra Sen
3. Voice of : Gopala Krishna (c) Henry Vivian Derozio
India Gokhale (d) Dwarkanath Tagore
Which of the pairs given above
is/are correctly matched?
19. Which of the following contribution are
(a) 1 and 2 only
(b) 2 and 3 only made by Bal Gangadhar Tilak to the cause of
(c) 1 only Indian National Movement?
(d) 2 only 1. Started the practice of using the
traditional religious festival to propagate
15. Tianwen 1 mission to Mars was recently nationalist ideas.
launched by 2. Associated with the Home Rule League
(a) European Union
movement.
(b) China
(c) Japan 3. Concluded the Lucknow Pact which
(d) South Korea provided for Hindu-Muslim unity in the
nationalist struggle.
16. In the last quarter of the nineteenth century, Select the correct answer using the code
which of the following were the demands of given below.
the members of the Indian National (a) 1 and 2 only
Congress?
(b) 2 and 3 only
1. Indianization of civil services
(c) 1 and 3 only
2. Reduction of import duties on textile
imports. (d) 1, 2 and 3
3. The right to bear arms.
Select the correct answer using the code 20. 'India Energy Modeling Forum' was recently
given below. launched by India jointly with
(a) 1 only
(a) Germany
(b) 1 and 3 only
(c) 2 and 3 only (b) Russia
(d) 1, 2 and 3 (c) Japan
(d) The United States of America
17. The main reason for the launch of Khilafat
movement was to: 21. What was the main demand of the Bardoli
(a) Demand for greater political Satyagraha organized under the leadership of
representation of Muslims in the
Vallabhbhai Patel?
legislature and executive.
(b) Agitate against the discrimination being (a) Writing off peasant debts
meted out to the Indian Muslims. (b) Granting land ownership right to tiller
(c) Protest against the treatment meted out (c) Released of the Political prisoner
to the Ottoman Empire by Britain. arrested during Non-Cooperation
(d) Show dissent against government’s
Movement
decision to go back on partition of
(d) Withdrawal of enhanced land revenue
Bengal.
4 www.visionias.in ©Vision IAS

Google it:- https://upscpdf.com


https://t.me/UPSC_PDF Download From > https://upscpdf.com https://t.me/UPSC_PDF

22. ASEEM platform, launched recently by the 25. In the context of Salt Satyagraha, consider
government, aims to the following statements:
1. Mahatma Gandhi broke the salt law at
(a) enable faster clearances of
Dandi.
environmental impact assessments of 2. K. Kelappan led a salt march to
infrastructure projects. Vedaranyam on the Tanjore coast.
(b) monitor real time power consumption 3. Sarojini Naidu participated in the non-
across Indian cities. violent march to Dharasana salt works.
Which of the statements given above is/are
(c) bridge demand-supply gap of skilled
correct?
workforce across sectors. (a) 3 only
(d) provide handholding services to attract (b) 1 and 2 only
foreign investment in strategic sectors. (c) 1 and 3 only
(d) 1, 2 and 3

23. Consider the following statements with


26. Consider the following pairs:
reference to police reforms during the Place in the news Located in
Governor Generalship of Lord Cornwallis: 1. Hagia Sophia : Turkey
1. Lord Cornwallis established circles or 2. Bhashan Char Island : Indonesia
3. Pasakha : Bhutan
thanas which were headed by European
Which of the pairs given above is/are
darogas.
correctly matched?
2. The zamindars were relieved of their (a) 1 only
police functions. (b) 2 and 3 only
3. In the villages, the policing duties were (c) 1 and 3 only
(d) 1, 2 and 3
continued by the village watchmen.
Which of the statements given above is/are
27. Which of the following leaders was the fist
correct? satyagrahi to offer Individual Satyagraha in
(a) 1 and 3 only 1940?
(b) 2 only (a) Vinoba Bhave
(b) Mahatma Gandhi
(c) 2 and 3 only
(c) Jawahar Lal Nehru
(d) 1, 2 and 3 (d) Brahma Dutt

24. Which of the following correctly describes 28. Which of the following countries are the
the term 'lunar maria', recently seen in the members of Quadrilateral Security Dialogue
news? (QUAD), often seen in the news?
1. India
(a) A thin layer of atmospheric gases
2. Japan
released by radioactive degassing. 3. Russia
(b) Craters and volcanic features on 4. USA
the surface of the moon. Select the correct answer using the code
given below.
(c) Optical halo rings around the moon
(a) 1, 2 and 4 only
formed by ice-crystal. (b) 2 and 3 only
(d) A short-lived change in the color and (c) 1 and 4 only
appearance of the Moon. (d) 1, 2, 3 and 4

5 www.visionias.in ©Vision IAS

Google it:- https://upscpdf.com


https://t.me/UPSC_PDF Download From > https://upscpdf.com https://t.me/UPSC_PDF

29. Consider the following passage: 33. Consider the following statements about
The nationalists were totally dissatisfied Public Safety Bill:
with the Act. They saw in it a mockery of
1. Its aim was to give the Government
their demands. They now demanded a
majority for non-official elected members power to deport 'undesirable' and
with the right to vote on the budget and, 'subversive' foreign nationals to stop the
thus, to the public purse. They raised the spread of socialist and communist ideas.
slogan ‘no taxation without representation.' 2. The bill was opposed by Swarijists but
Which of the following act is being referred
supported by the Capitalists &
to in the above passage?
(a) Indian Councils Act 1909 Liberals in the Legislative Council.
(b) Indian Councils Act of 1892 Which of the statements given above is/are
(c) Government of India Act 1919 correct?
(d) Indian Councils Act of 1861 (a) 1 only
(b) 2 only
30. PRAGYATA guidelines were recently
(c) Both 1 and 2
issued by the government in the context of
(a) facilitating digital education. (d) Neither 1 nor 2
(b) preventing cyber-attacks.
(c) regulating foreign funding. 34. Who of the following was/were economic
(d) incentivizing start-ups. critic of colonialism in India?
1. G.K. Gokhale
31. Consider the following statements about All-
2. G.V. Joshi
India Kisan Sabha:
1. All India Kisan Sabha (AIKS) was 3. Syed Ahmed Khan
established in 1936 under the Select the correct answer using the code
presidentship of Jawahar Lal Nehru. given below.
2. Faizpur Session, 1936-37 was the first (a) 1 only
session of Congress to be held in a
(b) 1 and 2 only
village, under the influence of AIKS.
Which of the statements given above is/are (c) 2 and 3 only
correct? (d) 1, 2 and 3
(a) 1 only
(b) 2 only
35. Arrange the following events of the Indian
(c) Both 1 and 2
(d) Neither 1 nor 2 freedom movement in correct chronological
sequence:
32. With reference to recently launched 1. Second Round Table Conference
Accelerate Vigyan Scheme, consider the 2. The foundation of All-India Students'
following statements: Federation
1. It aims at creating a single platform for
3. The first Independence (Swarajya) day
high-end scientific research by scholars
and students. was celebrated.
2. It is an initiative of NITI Aayog. Select the correct answer using the code
Which of the statements given above is/are given below.
correct? (a) 1-2-3
(a) 1 only
(b) 1-3-2
(b) 2 only
(c) Both 1 and 2 (c) 3-1-2
(d) Neither 1 nor 2 (d) 3-2-1

6 www.visionias.in ©Vision IAS

Google it:- https://upscpdf.com


https://t.me/UPSC_PDF Download From > https://upscpdf.com https://t.me/UPSC_PDF

36. Which of the following personalities were 39. MANODARPAN, an initiative by the
associated with the Mahalwari System in government, aims to
(a) provide forum for women to file real
India?
time complaint against cyber bullying.
1. Holt Mackenzie (b) provide counselling support to prisoners.
2. Robert Martins Bird (c) expedite development of digital
3. Francis Hastings infrastructure in underserved and remote
areas.
Select the correct answer using the code
(d) provide psychosocial support to students
given below. for their mental health and well-being.
(a) 1 and 2 only
(b) 2 and 3 only 40. With reference to the socio-political and
(c) 1 and 3 only educational organizations set up during the
colonial rule in India, consider the following
(d) 1, 2 and 3
pairs:
Organization Founder
37. With reference to the revolt of 1857,
1. Scientific Society : Syed Ahmed Khan
consider the following pairs:
2. East India : Dadabhai Nauroji
Leader Centre of the 1857 Association
Revolt 3. Madras Mahajan : Subramaniya Siva
1. Nana Saheb : Kanpur Sabha
2. Khan Bahadur : Faizabad Which of the pairs given above is/are
correctly matched?
3. Kunwar Singh : Bihar
(a) 1 only
4. Begum Hazrat : Lucknow (b) 1 and 2 only
Mahal (c) 2 and 3 only
Which of the pairs given above are correctly (d) 1, 2 and 3
matched?
41. In the 1930s, Karshak Sanghams was well
(a) 1, 3 and 4 only
known for leading peasant movements in
(b) 1 and 2 only which of the following regions?
(c) 2, 3 and 4 only (a) Malabar region
(d) 1, 2 , 3 and 4 (b) United Provinces
(c) Central Provinces
(d) Mysore
38. 'Cooling Emissions and Policy Synthesis
Report' was recently published by the United 42. Who among the following were the founders
Nations Environment Programme (UNEP) of the "Labor-Swaraj Party of Indian
and National Congress" established in 1925?
(a) Jawahar Lal Nehru, Subhash Chandra
(a) International Renewable Energy Agency
Bose and M.A.Ansari
(IRENA) (b) B. Krishna Pillai, E.M.S.
(b) World Economic Forum (WEF) Namboodiripad and K.C. George
(c) International Energy Agency (IEA) (c) Muzaffar Ahmed, Qazi Nazrul Islam and
Hemanta Kumar Sarkar
(d) Organization for Economic Co-
(d) Motilal Nehru, C.R. Das and Lala Lajpat
operation and Development (OECD)
Rai
7 www.visionias.in ©Vision IAS

Google it:- https://upscpdf.com


https://t.me/UPSC_PDF Download From > https://upscpdf.com https://t.me/UPSC_PDF

43. Consider the following statement with 45. With reference to Vaccine-derived
reference to the Government of India Act of polioviruses (VDPVs), consider the
following statements:
1858?
1. It is a rare strain of poliovirus that has
1. It is known as an Act for the Better genetically mutated from the strain
Government of India. contained in the oral polio vaccine.
2. The post of viceroy was abolished under 2. Currently, it is the only source of
this Act. polioviruses in the world.
3. It is an incurable form of poliovirus.
3. The secretary of state made under the
Which of the statements given above is/are
Act was directly responsible to the correct?
British Parliament. (a) 1 only
Which of the statements given above are (b) 2 and 3 only
(c) 3 only
correct?
(d) 1, 2 and 3
(a) 1 and 2 only
(b) 1 and 3 only 46. Consider the Gandhian movements given
(c) 2 and 3 only below and the associated Indian National
(d) 1, 2 and 3 Congress session in which the resolution for
the movement was passed:
Movement Session
44. Consider the following statements with
1.Non-Cooperation : Nagpur
reference to the Consumer Protection Act, Movement Session
2019: 2. Civil Disobedience : Karachi
1. The definition of consumer under the Movement Session
3. Quit India Movement : Calcutta
Act includes a person who obtains a
Session
good for resale or for any commercial Which of the pairs given above is/are
purpose. correctly matched?
2. It provides for product liability under (a) 1 and 2 only
(b) 2 and 3 only
which a product manufacturer or service
(c) 1 only
provider has to compensate a consumer
(d) 1, 2 and 3
for any harm caused by a defective good
or deficient service. 47. Who were called as the "responsivists"
3. The Act does away with Mediation as a during the Indian national movement?
(a) A group which was influenced by
mechanism of dispute resolution
socialist and communist ideas.
between parties.
(b) A group which wanted to organise an
Which of the statements given above is/are armed revolt against the colonial
correct? government.
(a) 1 only (c) A group which promoted communal
harmony.
(b) 2 and 3 only
(d) A group which offered support to the
(c) 2 only government for the protection of Hindu
(d) 1 and 2 only interests.

8 www.visionias.in ©Vision IAS

Google it:- https://upscpdf.com


https://t.me/UPSC_PDF Download From > https://upscpdf.com https://t.me/UPSC_PDF

48. He was a Vaislmava saint and poet, 51. Consider the following statements with
renowned for his devotion to Rama and as regard to Pitt's India Act, 1784:
the author of the Hindu epic
1. Under this Act, for the first time, the
Ramcharitmanas, retelling the Sansla t
Ramayana m Awadhi. He tenn 'British possessions in h1dia' was
contempora1y of Akbar. He is also ki10, used.
have sta1ted the Ramlila plays. The company monopoly of India.n a.nd
Which of the following saint 1s
Chinese trade remained intact.
described in the above passage?
3. For political matters related to India, a
(a) Valmiki
(b) Tulsidas Board of Control was created under the
(c) Ved Vyasa Act.
(d) Kamban Which -of the statements given above are
con·ect?
49. Which of the following statements
(a) 1 and 2 only
is not c-o!l'ect with regard to the Ryotwari
settlement? (b) 2 and 3 only
(a) It was intl'Oduced initially in the south (c) l . 2 a n d 3
and south-western India (d) 1 and 3 only
(b) The settlements tmder the system were
revised pet·iodically.
52. Which of the following is/are conect with
(c) The tax revenue was paid by the peasant
to the state through an inte1111edia1y. referenc.e to the Transgender Persons
(d) The system was introduced in 1820 by (Protection of Rights) Act, 2019?
Sir Thomas Munro. 1. It recognizes the right to self-perceived
gender identity and does away with the
50. Which of the following is/are con·ect with
need for certification from a District
reference to the Code on Wages Act, 2019?
1. It provides that different floor wages Magistrate.
may be fixed for different geographical 2. It offers reservations for the transgender
areas. under govenunent educational
2. It provides that employees working in
institutions and in public employment.
excess of a normal working day will be
entitled to overtime wage, which must 3. It provides for a National Com1cil for
be at least twice the nonnal rate of Transgender persons (NCT) with Union
wages. Minister of Social Jus6ce and
3. The wage period will be fixed by the Empowerment as Chairman.
respective state govemments as either -
Select the co!l'ect answer usmg the code
daily, weekly, fortnightly, or monthly.
Select the con-ect answer using the code given below.
given below. (a) 1 and 2 only
(a) 1 only (b) 3 only
(b) 2 and 3 only
(c) 1 and 3 only
(c) 1 and 2 only
(d) 1, 2 and 3 (d) 1, 2 and3

9 www.visionias.in ©Vision IAS

Google it:- https://upscpdf.com


https://t.me/UPSC_PDF Download From > https://upscpdf.com https://t.me/UPSC_PDF

Google it:- https://upscpdf.com


https://t.me/UPSC_PDF Download From > https://upscpdf.com https://t.me/UPSC_PDF

59. With reference to British rule in India, the 63. Ripon's resolution of 1882 pertained to:
'Downward filtration theory ' was used in the (a) Financial devolution to local bodies
context of which of the following?
(b) Improvement of labour conditions
(a) Army
(c) Shifting the capital of British India
(b) Police Reforms
(c) Health (d) Strictly regulating the vernacular press
(d) Education
64. Which of the following were part of the
60. This national park was first established as Gandhi-Irwin Pact of 1931?
Wildlife Sanctuary. It is a UNESCO World 1. Right to make salt for personal
Heritage Site. It is the biggest habitat of one-
consumption
horned rhinoceros. It was recently in news
2. Release of political prisoners who were
over the devastating floods causing the death
of many wild animals. not convicted for violence
Which of the following national parks is 3. Public inquiry into police excesses
being described in the above passage? 4. Right to peaceful picketing
(a) Manas National Park Select the correct answer using the code
(b) Valmiki National Park
given below.
(c) Kaziranga National Park
(a) 1 and 3 only
(d) Periyar National Park
(b) 3 and 4 only
61. Who among the following gave (c) 1, 2 and 4 only
the slogan ''no religion, no caste and no God (d) 1, 2, 3 and 4
for mankind''?
(a) Sri Narayana Guru 65. With reference to Vernacular Press Act
(b) Raja Rammohan Roy
(1878), which of the following statements
(c) Swami Dayanand
is/are correct?
(d) Sahadaran Ayyapan
1. The Act aimed to restrict the freedom of
62. Consider the following statements with both Indian language and English
reference to Revolt of 1857: newspapers in India.
1. Most of the rulers of princely states took 2. Under this Act the district magistrate
part in the revolt actively.
was all-empowered to take punitive
2. The revolt started in the form of a
actions, which cannot be appealed in the
mutiny in Meerut.
3. Bahadur Shah Zafar was proclaimed as court of law.
the Emperor of Hindustan during the 3. The Act was repealed by Lytton in 1882.
revolt. Select the correct answer using the code
Which of the statements given above are given below.
correct?
(a) 1 and 3 only
(a) 1 and 2 only
(b) 2 only
(b) 2 and 3 only
(c) 1 and 3 only (c) 2 and 3 only
(d) 1, 2 and 3 (d) 1, 2 and 3

11 www.visionias.in ©Vision IAS

Google it:- https://upscpdf.com


https://t.me/UPSC_PDF Download From > https://upscpdf.com https://t.me/UPSC_PDF

66. Battle of Buxar was one of the most decisive 69. Which of the following statements is/are
battles of Indian history. In this context, correct regarding the Congress Benares
which of the following are correct with session of 1905 ?
reference to consequences of Battle of 1. It was presided by Dadabhai Naoroji.
Buxar? 2. Self government or Swaraj was declared
1. The Company secured the diwani rights as the goal of Congress at this session.
of entire Mughal India. 3. At the session it was decided to extend
2. Bengal came under dual system of the Swadeshi movement outside Bengal.
administration. Select the correct answer using the code
3. The battle ended with the signing of the given below.
Treaty of Allahabad. (a) 1 only
Select the correct answer using the code (b) 1 and 2 only
given below. (c) 2 and 3 only
(a) 1 and 2 only (d) None
(b) 2 and 3 only
(c) 1 and 3 only 70. Which of the following best describes the
(d) 1, 2 and 3 strategy of Indian national movement from
1920s to 1940s?
67. Rogan painting refers to: (a) Struggle-truce-struggle
(a) a hand-painted fabric art in the Kutch (b) Struggle till the freedom
region of India. (c) Struggle within the constitutional sphere
(b) a miniature art practiced in Rajasthan. (d) Struggle without harming the British
(c) ritualistic mural art practiced by local rule
tribal women in Jharkhand.
(d) a three-color cotton painting 71. With reference to the recently launched
in Telangana. National Initiative for School Heads' and
Teachers' Holistic Advancement
68. Consider the following statements with (NISHTHA), consider the following
reference to Federation of Indian Chambers statements:
of Commerce and Industry (FICCI): 1. It is a capacity-building programme for
1. It was established through the efforts of improving the learning outcomes of
Indian capitalists like G.D. Birla students through the integration of
and Purshottamdas Thakurdas. technology.
2. The FICCI remained as a trade union 2. It has been initiated by the National
organization and effectively restricted Council of Educational Research and
itself from intervening in politics. Training.
Which of the statements given above is/are Which of the statements given above is/are
correct? correct?
(a) 1 only (a) 1 only
(b) 2 only (b) 2 only
(c) Both 1 and 2 (c) Both 1 and 2
(d) Neither 1 nor 2 (d) Neither 1 nor 2

12 www.visionias.in ©Vision IAS

Google it:- https://upscpdf.com


https://t.me/UPSC_PDF Download From > https://upscpdf.com https://t.me/UPSC_PDF

72. With reference to the MSME Emergency 75. "The movement occurred in the Eastern
Response Programme, recently in news, Bengal. The peasants organised agrarian
consider the following statements: leagues to resist increased rents by
1. It is an initiative by the International zamindars. The main form of struggle was
Monetary Fund. legal resistance. After the uprising, the
2. It aims to increase the flow of finance Bengal Tenancy Act, 1885 was passed to
into the hands of MSMEs which are protect the tenant's rights."
severely impacted by the COVID-19 Which of the following movements is being
crisis. described in the above-given passage?"
Which of the statements given above is/are
(a) Tebhaga movement
correct?
(b) Pabna revolt
(a) 1 only
(c) Indigo revolt
(b) 2 only
(d) Eka movement
(c) Both 1 and 2
(d) Neither 1 nor 2
76. Which of the following organizations/
movements was/were reformist in nature?
73. Which of the following correctly describes
1. Tatvabodhini Sabha
the phenomenon of 'Helium Flash'?
2. Rehnumai Mazdayasan Sabha
(a) The illuminated tail of a comet when it
3. Deoband Movement
passes through the inner Solar System.
Select the correct answer using the code
(b) A thermonuclear phenomenon to
given below.
generate power from nuclear fusion
(a) 1 only
reactions.
(b) 1 and 2 only
(c) The natural light display caused by solar
(c) 2 and 3 only
wind disturbances in the magnetosphere.
(d) A lithium enrichment stage in the (d) 1, 2 and 3

evolution of star caused by the rise in


77. ‘'He was the Principal of the Sanskrit
temperature and pressure.
College in Bengal. He raised his voice in

74. He joined Anushilan Samiti and was tried in favor of the Widow Remarriage Act 1855.

the Lahore Conspiracy Case. He died after As a Government Inspector of schools, he


his 63 days prolonged hunger strike organized thirty-five girl’s schools.’'
demanding better treatment for political The above passage refers to which of the
prisoners '. He was following social reformers?
(a) Jatin Das (a) Debendranath Tagore
(b) Batukeshwar Dutt (b) Ishwar Chandra Vidyasagar
(c) Surya Sen (c) Jagan-nath Shankar Seth
(d) Sachindra Nath Sanyal (d) Karsondas Mulji

13 www.visionias.in ©Vision IAS

Google it:- https://upscpdf.com


https://t.me/UPSC_PDF Download From > https://upscpdf.com https://t.me/UPSC_PDF

78. The 1907 Surat session of Indian National 82. Which of the following events happened in
Congress is significant in the history of the the Lahore session of the Indian National
freedom movement, because: Congress (1929)?
(a) Congress accepted separate electorate 1. Resolution on National Economic
Programme
for Muslims.
2. Resolution to reject the two-nation
(b) Congress split into moderate and
theory
extremist groups. 3. Poorna Swaraj Declaration
(c) Swadeshi movement was launched. 4. Resolution on Fundamental rights
(d) "Swaraj" was adopted as the goal of the Select the correct answer using the code
Indian National Congress. given below.
(a) 1 and 2 only
79. Which of the following leaders founded the (b) 3 only
(c) 3 and 4 only
"Harijan Sevak Sangh" in 1932?
(d) 1, 3 and 4 only
(a) Mahatma Gandhi
(b) Dr. B.R. Ambedkar
83. In the context of Indigo revolt of the 19th
(c) Jawahar Lal Nehru
century, consider the following statements:
(d) K. Kelappan 1. The revolt occurred in the western
regions of the United Provinces.
80. Consider the following statements with 2. The rebellion against the European
respect to the Simon Commission: planters later turned into a movement
1. No Indian was a member of the Simon against the colonial government.
Commission. 3. The Indigo Commission appointed to
inquire into the problem, found that the
2. The report of the Simon Commission
indigo cultivation system is oppressive.
recommended for the Dominion Status
Which of the statements given above is/are
for India. correct?
Which of the statements given above (a) 1 and 2 only
is/are correct? (b) 3 only
(a) 1 only (c) 2 and 3 only
(b) 2 only (d) 1 and 3 only
(c) Both 1 and 2
(d) Neither 1 nor 2 84. Consider the following statements regarding
the Congress sessions and princely states'
participation in freedom struggle:
81. Consider the following statements with
1. The Nagpur Session of 1920 allowed the
reference to the changes in the army after
people of the States for the first time to
1858: initiate political activities in the name of
1. The proportion of the Europeans to Indian National Congress.
Indians in the army was reduced. 2. The Haripura Session of 1938 extended
2. The European troops were kept in key the call for freedom struggle to the
geographical and military positions. people of the States.
Which of the statements given above is/are Which of the statements given above is/are
correct?
correct?
(a) 1 only
(a) 1 only
(b) 2 only
(b) 2 only (c) Both 1 and 2
(c) Both 1 and 2 (d) Neither 1 nor 2
(d) Neither 1 nor 2

14 www.visionias.in ©Vision IAS

Google it:- https://upscpdf.com


https://t.me/UPSC_PDF Download From > https://upscpdf.com https://t.me/UPSC_PDF

85. With reference to disqualification of 88. During the Civil Disobedience Movement, a
legislators under the anti-defection law, powerful agitation against 'Cunningham
recently seen in the news, consider the Circular ' was launched in Assam because it
following statements: (a) imposed censorship on national and
1. When a member is disqualified under
regional press.
the anti-defection law, he can be
(b) forbade industrial workers to form trade
immediately appointed a minister and
unions.
will get six months to get re-elected.
(c) prohibited students from participating in
2. Resigning as an MLA and subsequently
joining another party does not attract political activities.
disqualification. (d) provided for imprisonment without trial.
Which of the statements given above is/are
correct? 89. Which of the following parties were a part of
(a) 1 only Boycott of the Simon Commission in 1928?
(b) 2 only 1. Liberal Federation
(c) Both 1 and 2 2. Bahishkrita Hitakarini Sabha
(d) Neither 1 nor 2
3. Indian Industrial and Commercial
Congress
86. Consider the following statements regarding
4. Hindu Mahasabha
the Battle of Plassey:
Select the correct answer using the code
1. It was fought with British East India
Company on one side and the Nawab of given below.
Bengal and French force on the other (a) 1 and 2 only
side. (b) 2 and 3 only
2. After the battle, Mir Jafar was made the (c) 1, 3 and 4 only
Nawab of Bengal. (d) 1, 2, 3 and 4
Which of the statements given above is/are
correct? 90. Portuguese were the first to arrive in India
(a) 1 only and had established monopoly of eastern
(b) 2 only
trade for nearly a century. Yet they were
(c) Both 1 and 2
unable to maintain dominions in India
(d) Neither 1 nor 2
because:
1. Their merchants enjoyed less power than
87. Which of the following was/were the
demands of the Nehru Report of 1928? landed aristocrats.
1. It sought reservations for Muslims in all 2. As compared to other European powers
the provincial governments. they lagged in the development of
2. It recommended universal adult suffrage shipping.
and dissociation of state from religion. 3. They followed a policy of religious
3. It demanded complete independence for intolerance.
India. Select the correct answer using the code
Select the correct answer using the code given below.
given below.
(a) 1 and 2 only
(a) 1 and 2 only
(b) 2 and 3 only
(b) 2 only
(c) 1 and 3 only
(c) 1 and 3 only
(d) 1, 2 and 3 (d) 1, 2 and 3

15 www.visionias.in ©Vision IAS

Google it:- https://upscpdf.com


https://t.me/UPSC_PDF Download From > https://upscpdf.com https://t.me/UPSC_PDF

91. During the nineteenth century in British 93. Bhagat Singh, a revolutionary socialist
India, which of the following objectives leader, was associated with which of the
following organizations?
guided the colonial economic policies?
1. Hindustan Socialist Republican
1. Transformation of colonies into a Association
supplier of foodstuffs and raw materials 2. Naujawan Bharat Sabha
3. Workers' & Peasants Party
to the metropolis.
Select the correct answer using the code
2. A market for the metropolitan given below.
manufacturers (a) 1 only
(b) 2 and 3 only
3. A field for the investment of British
(c) 1 and 2 only
capital. (d) 1, 2 and 3
Select the correct answer using the code
given below. 94. With reference to the recently launched
scheme 'Agriculture Infrastructure Fund',
(a) 1 and 2 only
consider the following statements:
(b) 2 and 3 only 1. It aims to finance projects related to the
(c) 1, 2 and 3 improvement of post-harvesting
infrastructure.
(d) 1 only
2. It is a centrally sponsored scheme.
Which of the statements given above is/are
92. Consider the following statements with correct?
regard to civil services in India: (a) 1 only
(b) 2 only
1. Lord Cornwallis introduced the
(c) Both 1 and 2
covenanted civil services and the (d) Neither 1 nor 2
uncovenanted civil services.
2. Lord Wellesley established the college 95. With reference to Anti-Chowkidara
Movement, consider the following
of Fort William at Calcutta to educate
statements:
recruits for civil services 1. Anti-Chowkidara movement became a
3. The Charter Act of 1853 provided for an part of the Civil Disobedience
Movement.
open competitive examination for the
2. Chowkidara was a tax levied on the
recruitment of civil servants. villages by the Panchayats.
Which of the statements given above is/are 3. It was prominent in various parts of
Southern & North Western India.
correct?
Which of the statements given above are
(a) 1 and 3 only correct?
(b) 2 only (a) 1 and 2 only
(c) 1 only (b) 2 and 3 only
(c) 1 and 3 only
(d) 1, 2 and 3
(d) 1, 2 and 3

16 www.visionias.in ©Vision IAS

Google it:- https://upscpdf.com


https://t.me/UPSC_PDF Download From > https://upscpdf.com https://t.me/UPSC_PDF

96. Which of the following is/are correct with 99. Consider the following statements:
reference to the UN Convention Against 1. The early nationalists actively demanded
Torture (UNCAT)?
industrial worker's rights.
1. It prevents the extradition of criminals to
a country where there is a danger of 2. Sasipada Banerjee brought out the
torture. journal Bharat Sramjeebi to educate the
2. India has signed and ratified the
workers.
convention.
Select the correct answer using the code 3. Narayan Meghajee Lokhanday started
given below. the Bombay Mill and
(a) 1 only
Millhands’Association in 1890.
(b) 2 only
(c) Both 1 and 2 Which of the statements given above is/are
(d) Neither 1 nor 2 correct?
(a) 3 only
97. In the context of the Indian freedom
movement, Khudai Khidmatgars organised (b) 2 and 3 only
(a) a peasant movement in Punjab (c) 1 and 2 only
(b) a movement for upliftment of tribals in
(d) 1, 2 and 3
Gujarat
(c) non violent resistance movement in
North-West Frontier Province 100. The ‘Post War Economic Development
(d) international political movement to Committee,’ set up in 1942 drafted the
overthrow British rule in India.
Bombay Plan. The Bombay Plan provided

98. With reference to Active Pharmaceutical for which of the following?


Ingredients (API), consider the following 1. Comprehensive land reform
statements:
2. Cooperativization of production
1. API is a biologically active component
of a drug product required for making 3. Series of workers’ welfare schemes
medicines. 4. Complete nationalization
2. India is the largest producer and exporter
Select the correct answer using the code
of APIs in the world.
Which of the statements given above is/are given below.
correct? (a) 1, 2 and 3 only
(a) 1 only (b) 1 and 3 only
(b) 2 only
(c) 2 and 4 only
(c) Both 1 and 2
(d) Neither 1 nor 2 (d) 1, 2, 3 and 4

17 www.visionias.in ©Vision IAS

Google it:- https://upscpdf.com


https://t.me/UPSC_PDF Download From > https://upscpdf.com https://t.me/UPSC_PDF

VISIONIAS
www.visionias.in
ANSWERS & EXPLANATIONS
GENERAL STUDIES (P) TEST – 3183 (2021)

Q 1.B
• The idea of the Indian National Army (INA) was first conceived in Malaya by Mohan Singh, an
Indian officer of the British Indian Army. Hence statement 1 is correct.
• Its aim was to secure India's independence from British rule. It formed an alliance with Imperial
Japan in the latter's campaign in the Southeast Asian theatre of WW-II.
• Indian prisoners of war were handed over by the Japanese to Mohan Singh who then tried to
recruit them into an Indian National Army. By the end of 1942, forty thousand men expressed their
willingness to join the INA. The outbreak of the Quit India Movement gave a fillip to the INA as well.
Anti-British demonstrations were organized in Malaya. On 1st September 1942, the first division of the
INA was formed with 16,300 men.
• This first INA collapsed and was disbanded in December that year after differences between the INA
leadership and the Japanese military over its role in Japan's war in Asia. Rash Behari Bose handed over
INA to Subhas Chandra Bose.
• The second phase of the INA began when Subhas Chandra Bose was brought to Singapore on 2 July
1943, by means of German and Japanese submarines. He went to Tokyo and Prime Minister Tojo who
declared that Japan had no territorial designs on India. Bose returned to Singapore and set up the
Provisional Government of Free India on 21 October 1943. The Provisional Government then declared
war on Britain and the United State and was recognised by the Axis powers and their satellites. He
declared the INA as the army of Arzi Hukumat-e-Azad Hind (Provisional Government of Free
India).
• Subhas Bose set up two INA headquarters, in Rangoon and in Singapore, and began to reorganize
the INA. Hence statement 3 is not correct.
• Recruits were sought from civilians, funds were gathered, and even a women’s regiment called the
Rani Jhansi regiment was formed. Hence statement 2 is not correct.
• This second INA fought along with the Imperial Japanese Army against the British and Commonwealth
forces in the campaigns in Burma: at Imphal and Kohima, and later against the Allied retaking of Burma
during 1942-43 but failed to achieve independence.

Q 2.C
• The Congress Socialist Party (CSP) was a socialist caucus within the Indian National Congress. It was
founded in 1934 by Congress members who rejected what they saw as the anti-rational mysticism of
Gandhi as well as the sectarian attitude of the Communist Party of India towards the Congress.
• Influenced by Fabianism as well as Marxism-Leninism, the CSP included advocates of armed struggle or
sabotage (such as Yusuf Meherally, Jai Prakash Narayan, and Basawon Sinha) as well as those who
insisted upon ahimsa or Non-violent resistance (such as Acharya Narendra Deva).
• The CSP advocated decentralized socialism in which co-operatives, trade unions, independent farmers,
and local authorities would hold a substantial share of the economic power.
• It was founded in 1934, in Bombay, by Jaiprakash Narayan, Minoo Masaani, Ram Manohar Lohia
and Acharya Narendra Dev. Hence statement 1 is correct.
• From the beginning, all the Congress Socialists were agreed upon basic propositions:
o that the primary struggle in India was the national struggle for freedom and that nationalism was a
necessary stage on the way to socialism;
o that socialists must work inside the National Congress because it was the primary body leading
the national struggle; Hence statement 2 is correct.
o that to achieve this objective they must organize the workers and peasants in their class organizations,
wage struggles for their economic demands and make them the social base of the national struggle.
1 www.visionias.in ©Vision IAS

Google it:- https://upscpdf.com


https://t.me/UPSC_PDF Download From > https://upscpdf.com https://t.me/UPSC_PDF

• Thus the CSP from the beginning assigned itself the task of both transforming the Congress and of
strengthening it.
• In 1936, the CSP began fraternal relations with the Lanka Sama Samaja Party of Ceylon. In 1937 the CSP
sent Kamaladevi Chattopadhyaya on a speaking tour of the island.
• Members of the CSP were particularly active in the Quit India Movement of August 1942.
• After independence, the CSP broke away from Congress, under the influence of Jayaprakash Narayan,
and Lohia to form the Socialist Party of India.

Q 3.D
• On August 1935, the Government of India passed the longest act i.e. Government of India Act 1935
under the British Act of Parliament. This act also included the Government of Burma Act 1935.
• The Act provided for the establishment of an All-India Federation to be based on the union of the
British Indian provinces and the Princely States. Hence statement 1 is correct.
o The representatives of the States to the federal legislature were to be appointed directly by the Princes
who were to be used to check and counter the nationalists.
o India would become a federation if 50% of Indian states decided to join it. They would then have a
large number of representatives in the two houses of the central legislature.
o However, the provisions with regards to the federation were not implemented as princes refused to
join the federation.
• The franchise was limited to about one-sixth of the adults.
• Defence and foreign affairs would remain outside the control of the federal legislature, while the Viceroy
would retain special control over other subjects.
• The Act divided the powers between the Centre and Units in terms of 3 lists: Federal List (for Centre, with
59 items), Provincial List (for provinces, with 54 items) and the Concurrent List (for both, with 36
items). Residuary powers were given to the Viceroy. Hence statement 3 is correct.
• It provided for the adoption of dyarchy at the Centre. Consequently, the federal subjects were divided
into reserved subjects and transferred subjects. However, this provision of the Act did not come into
operation at all.
• The provinces were to be governed under a new system based on provincial autonomy under which
elected ministers controlled all provincial departments. Dyarchy which was introduced at the
provincial level in Government of India Act, 1919 was abolished. Hence statement 2 is correct.
• It abolished the Council of India, established by the Government of India Act of 1858. The Secretary of
State for India was provided with a team of advisors.
• The appointment of the Viceroy and Governors remained in the hands of the British government and they
were not responsible to the legislatures.
• It introduced bicameralism in six out of eleven provinces. Thus, the legislatures of Bengal, Bombay,
Madras, Bihar, Assam and the United Provinces were made bicameral consisting of a legislative council
(upper house) and a legislative assembly (lower house).
• Once again, the Governors, appointed by the British Government, retained special powers. They could
veto legislative and administrative measures, especially those concerning minorities, the rights of civil
servants, law and order and British business interests.
• The Governor also had the power to take over and indefinitely run the administration of a province. Thus
both political and economic power remained concentrated in British hands; colonialism remained intact.
• It provided for the establishment of a Reserve Bank of India to control the currency and credit of the
country.
• It provided for the establishment of not only a Federal Public Service Commission but also a Provincial
Public Service Commission and Joint Public Service Commission for two or more provinces.
• It provided for the establishment of a Federal Court, which was set up in 1937.

Q 4.A
• The Chiang Mai Initiative (CMI) was the first regional currency swap arrangement launched by the
ASEAN+3 countries in May 2000 at an annual meeting of the Asian Development Bank to address the
short-term liquidity difficulties in the region and to supplement the existing international financial
arrangements. CMI is composed of: (a) the ASEAN Swap Arrangement (ASA) among ASEAN countries,
and (b) a network of bilateral swap arrangements (BSAs) among the ASEAN+3 countries. Hence, option
(a) is correct.
• In 2004, the ASEAN+3 countries proposed to have a more advanced framework for liquidity support
which focuses on multilateralization of the CMI (Chiang Mai Initiative Multilateralization - CMIM). The
CMIM is a multilateral currency swap arrangement among the ten (10) members of the Association of

2 www.visionias.in ©Vision IAS

Google it:- https://upscpdf.com


https://t.me/UPSC_PDF Download From > https://upscpdf.com https://t.me/UPSC_PDF

Southeast Asian Nations (ASEAN), the People’s Republic of China (including Hong Kong), Japan, and
South Korea.
• The core objectives of the CMIM are (i) to address balance-of-payments and short-term liquidity
difficulties in the region and (ii) to supplement the existing international financial arrangements.
• Asian multilateralism has often been born out of crises. The Chiang Mai Initiative emerged in the
aftermath of the late 1990s financial crisis. China’s post-COVID aggression seems to be reshaping
Asia. The Indian, Japanese, Malaysian, and Australian governments have all taken concrete steps to
reduce their economic exposure to Beijing, spanning investment, manufacturing, and technology. The
Quadrilateral Dialogue between Australia, India, Japan, and the United States is growing stronger and
even expanding.

Q 5.B
• As the world war situation worsened in 1941-42, President Roosevelt of the USA and President Chiang
Kai-Shek of China as also the Labour Party leaders of Britain put pressure on Churchill to seek the active
cooperation of Indians in the War. To secure this cooperation the British Government sent to India in
March 1942 a mission headed by a Cabinet minister Stafford Cripps, a left-wing Labourite who had
earlier actively supported the Indian national movement.
• Even though Cripps announced that the aim of British policy in India was ‘the earliest possible realization
of self- government in India,’ the Draft Declaration he brought with him was disappointing.
• The Declaration promised India Dominion Status and a constitution-making body after the second
world war whose members would be elected by the provincial assemblies and nominated by the rulers in
case of the princely states. Hence statements 2 is correct.
• The Pakistan demand was accommodated by the provision indirectly that any province which was not
prepared to accept the new constitution would have the right to sign a separate agreement with
Britain regarding its future status. For the present, the British would continue to exercise sole control over
the defence of the country. Hence statement 1 is not correct.
• Negotiations between Cripps and the Congress leaders broke down. The Congress objected to the
provision for Dominion Status rather than full independence, the representation of the princely states in
the constituent assembly not by the people of the states but by the nominees of the rulers, and above all by
the provision for the partition of India.
• The British Government also refused to accept the demand for the immediate transfer of effective power
to the Indians and for a real share in the responsibility for the defence of India. An important reason for
the failure of the negotiations was the incapacity of Cripps to bargain and negotiate.

Q 6.A
• 1937 elections: Provincial elections were held in British India in the winter of 1936-37 as mandated
by the Government of India Act 1935. Elections were held in eleven provinces - Madras, Central
Provinces, Bihar, Orissa, United Provinces, Bombay Presidency, Assam, NWFP, Bengal, Punjab
and Sindh. As the federal part did not come into existence after the Government of India Act, 1935 due to
reservations of princely states, no elections were held at the federal Level. Hence statement 1 is correct.
• The Congress decided at Lucknow in early 1936 and at Faizpur in late 1936 to fight the elections and
postpone the decision on office acceptance to the post-election period.
• The Congress won a massive mandate at the polls despite the narrow franchise. It won 716 out of 1,161
seats it contested. It had a majority in most of the provinces. The exceptions were Bengal, Assam,
the NWPF, Punjab and Sind; and in the first three, it was the largest single party. The prestige of the
Congress as the alternative to the colonial state rose even higher.
• After a few months’ tussles with the Government, the Congress Working Committee decided to accept
office under the Act of 1935. During July, it formed Ministries in six provinces: Madras, Bombay,
Central Provinces, Orissa, Bihar and U.P. Later, Congress Ministries were also formed in the
North-West Frontier Province and Assam. The Non-Congress Ministries were formed in Bengal
(Krishi Praja Party with the support of Muslim League and Independent Muslims), Punjab
(Unionist Party) & Sindh (United Sindh Party). Hence statement 2 is not correct.
• To guide and coordinate their activities and to ensure that the British hopes of the provincialization of the
Congress did not materialize, a central control board known as the Parliamentary Sub-Committee
was formed, with Sardar Patel, Maulana Abul Kalam Azad and Rajendra Prasad as members.
• The Congress was now to function both as a government in the provinces and as the opposition vis-a-vis
the Central Government where effective state power lay. It was to bring about social reforms through the
legislature and administration in the provinces and at the same time carry on the struggle for independence
and prepare the people for the next phase of mass struggle.
3 www.visionias.in ©Vision IAS

Google it:- https://upscpdf.com


https://t.me/UPSC_PDF Download From > https://upscpdf.com https://t.me/UPSC_PDF

Q 7.B
• Recently, several environmentalists and pro-GM farmers' organisations have asked the government
to take concrete stand on GM crops. In the absence of clear directions, a black market has
developed for various unapproved GM crops.
• Genetically modified crops (GM crops) are plants used in agriculture, the DNA of which has been
modified using genetic engineering methods. Usually, the cells are then grown in tissue culture where
they develop into plants. The seeds produced by these plants will inherit the new DNA. In most cases, the
aim is to introduce a new trait to the plant which does not occur naturally in the species.
• Development, cultivation and transboundary movement of GM crops is regulated so as to ensure the
safety of animal health, human health, and biodiversity.
• All activities involving research and development of products containing GMOs including transgenic
crops, pharma products, industrial products, food, and foodstuffs are regulated by the Genetic Engineering
Appraisal Committee (GEAC). It is under the purview of the Ministry of Environment, Forest and
Climate Change (MoEF&CC). Hence, statement 1 is not correct.
• Currently Bt Cotton is the only GM crop allowed for commercial cultivation in India. However, GM
variants of crops like brinjal, soya, etc are cultivated and circulated in black markets. Hence, statement 2
is correct.

Q 8.B
• The political condition and the economic trend of the 19th century India induced the British to construct
railways all over India. Railways, it was believed, would assist the economic development of India and
provide both a market for British goods and a source of raw materials. It would also be helpful in the
administration and protection of India by facilitating the movement of troops within the subcontinent. But
their hidden or the main objective for the introduction of Railways was exploitation of the natural
resources in India and the consolidation of the British empire. Hence statement 1 is not correct.
• The Indian Post Office was established in 1837, Asia’s first adhesive stamp, the Scinde Dawk, was
introduced in 1852 by Sir Bartle Frere, the British East India Company’s administrator of the province of
Sind. The first stamps valid for postage throughout India were placed on sale in October, 1854. The new
system was recommended by the Governor General, Lord Dalhousie and adopted by the East India
Company’s Court of Directors. Hence statement 2 is correct. It introduced “low and uniform” rates
for sending mail efficiently throughout the country within the jurisdiction of the East India Company.
• Telegraph services in India date back to 1850. The first telegraph line, though only experimental was
established between Calcutta and Diamond Harbour. Hence statement 3 is correct. The British East
India Company started using the telegraph in 1851 and till 1854 telegraph lines were laid across the
country.

Q 9.A
• All India States' People's Conference was a national-level body founded in 1927 with its headquarters
at Bombay with the objective to coordinate political activities in different states and also to raise
moderate demands for democratic rights and constitutional changes in Princely States. Hence,
statement 1 is correct and statement 2 is not correct.
• The men chiefly responsible for its formation were Balwantrai Mehta, Manikial Kothari and G.R.
Abhayankar. It was an independent organisation and not formed by Indian National Congress.
• In 1939, the AISPC elected Jawaharlal Nehru as its President for the Ludhiana session and remained so
until 1946.

Q 10.D
• Gopal Hari Deshmukh, was an Indian activist, thinker, social reformer, and writer from Maharashtra. At
age 25, Deshmukh started writing articles aimed at social reform in Maharashtra in the
weekly Prabhakar under the pen name Lokhitawadi.
• He promoted emancipation (liberation) and education of women and wrote against arranged child
marriages, dowry system, and polygamy, all of which were prevalent in India in his times.
• He wrote against the evils of the caste system, condemned harmful Hindu religious orthodoxy, and
attacked the monopoly in religious matters and rituals which Brahmin priests had through a long tradition
(Deshmukh, himself, belonged to the Brahmin caste). He enunciated certain 15 principles for bringing
about religious reform in Hindu society. For all his social works he came to be known as Lokhitwadi
(one who works in the interest of the people). Hence option (d) is the correct option.

4 www.visionias.in ©Vision IAS

Google it:- https://upscpdf.com


https://t.me/UPSC_PDF Download From > https://upscpdf.com https://t.me/UPSC_PDF

• He took a leadership role in founding Gyan Prakash, Indu Prakash, and Lokhitwadi periodicals in
Maharashtra.
• In 1867, the government appointed him a small cause judge in Ahmedabad, Gujarat. He worked as a
Diwan also in Ratlam state. The government had commended him with the honorifics 'Justice of
Peace' and 'Raobahadur' while he was still working. He retired as a sessions judge. He held many other
important positions, including those of the Assistant Inam Commissioner, Joint Judge of Nasik High
Court, and Member of the Law Council.

Q 11.D
• The colonial intrusion and the triumvirate of trader, moneylender and revenue farmer in sum
disrupted the tribal identity. As a result the tribal people, spread over a large part of India, organized
hundreds of militant outbreaks and insurrections during the 19th century.
• Santhal rebellion (1855-56): Continued oppression of the Santhals, an agricultural people, who had fled
to settle in the plains of the Rajmahal hills (Bihar) led to the Santhal rebellion against the zamindars.
The money-lenders who had the support of the police among others had joined the zamindars to subject
the peasants to oppressive exactions and dispossession of lands. The rebellion turned into an anti-British
movement. Under Sidhu and Kanhu, two brothers, the Santhals proclaimed an end to Company rule,
and declared the area between Bhagalpur and Rajmahal as autonomous. The rebellion was
suppressed by 1856.
• Munda rebellion (Ulgulan) under Birsa Munda: In 1899-1900, the Mundas in the region south of
Ranchi rose under Birsa Munda. The rebellion which began as a religious movement gathered political
force to fight against the introduction of feudal, zamindari tenures, and exploitation by money-lenders and
forest contractors. On Christmas Eve, 1899, Birsa proclaimed a rebellion to establish Munda rule in
the land and encouraged ‘the killing of thikadars and jagirdars and Rajas and Hakims (rulers) and
Christians.’ He was, however, captured in the beginning of February 1900 and he died in jail in June. The
rebellion had failed.
• The Kols of Chhotanagpur rebelled from 1820 to 1837. It was a revolt of the indigenous Kol people as
a reaction to unfair treatment brought on by the systems of land tenure and administration that had been
introduced by British powers in the area. Thousands of them were massacred before British authority
could be re-imposed.
• Hence the correct order is 3-1-2 and option (d) is the correct answer.

Q 12.B
• The revolutionary activity in 1920s was dominated by the Hindustan Republican Association/Army or
HRA (later renamed Hindustan Socialist Republican Association or HSRA). The HRA was founded in
October 1924 in Kanpur by Ramprasad Bismil, Jogesh Chandra Chatterjee and Sachin
Sanyal, with an aim to organise an armed revolution to overthrow the colonial government. The most
important action of the HRA was the Kakori robbery in 1925. The men held up the 8-Down train at
Kakori and looted its official railway cash. The Government arrested a large number of young men and
tried them in the Kakori Conspiracy Case. Ashfaqulla Khan, Ramprasad Bismil, Roshan Singh and
Rajendra Lahiri were hanged, four others were sent to the Andamans for life and seventeen others were
sentenced to long terms of imprisonment. Hence pair 2 is correctly matched.
• Lala Lajpat Rai died due to lathi blows received during a lathi- charge on an anti-Simon Commission
procession. Later Bhagat Singh, Azad and Rajguru dead Saunders, the police official responsible for the
lathi charge in Lahore. Bhagat Singh, Sukhdev and Rajguru were tried in the Lahore conspiracy
case. Hence pair 1 is correctly matched.
• The rapid growth of communist and The Workers and Peasants Party (WPP) influence over the national
movement in 1920s was checked by British by two developments. One was the severe repression to which
Communists were subjected by the Government. Second, in 1924, the Government had tried to cripple the
nascent communist movement by trying S.A. Dange, Muzaffar Ahmed, Nalini Gupta and Shaukat
Usmani in the Kanpur Bolshevik Conspiracy Case. All four were sentenced to four years of
imprisonment. Hence pair 3 is not correctly matched.

Q 13.D
• In October 1929 Lord Irwin, the Viceroy stated that it is implicit in the Declaration of 1917 that the
natural issue of India’s progress as there contemplated, is the attainment of dominion status.’
• He also promised a Round Table Conference as soon as the Simon Commission submitted its report.
• Two days later, on November 2, 1929, a conference of prominent national leaders issued a ‘Delhi
Manifesto’ in which they demanded that it should be made clear that the purpose of the Round Table
5 www.visionias.in ©Vision IAS

Google it:- https://upscpdf.com


https://t.me/UPSC_PDF Download From > https://upscpdf.com https://t.me/UPSC_PDF

Conference was not to discuss when Dominion Status should be granted but to formulate a scheme for
its implementation. They also demanded that Congress should have majority representation at the
conference; and there should be a general amnesty for political prisoners and a policy of conciliation.
• Hence option (d) is the correct answer.

Q 14.C
• The Press was the chief instrument for carrying out the task for arousing, training, mobilizing and
consolidating nationalist public opinion. Powerful newspapers emerged during these years under
distinguished and fearless journalists.
• These were the Hindu and Swadesamitran under the editorship of G. Subramaniya Iyer, Kesari and
Mahratta under B.G. Tilak, Bengalee under Surendranath Banerjea, Amrita Bazar Patrika under Sisir
Kumar Ghosh and Motilal Ghosh, Sudharak under G.K. Gokhale, Indian Mirror under N.N.
Sen, Voice of India under Dadabhai Naoroji, Hindustani and Advocate under G.P. Varma and Tribune
and Akhbar-i-Am in Punjab, Indu Prakash, Dnyan Prakash, Kal and Gujarati in Bombay, and Som
Prakash, Banganivasi, and Sadharani in Bengal. Hence, only pair 1 is correctly matched.

Q 15.B
• China recently launched its first successful Mars probe, called Tianwen-1 (formerly Huoxing 1),
which means “Questions to Heaven”. Hence option (b) is the correct answer.
o The spacecraft consists of an orbiter, a lander and a rover.
o The spacecraft, weighing 5 tonnes, launched on a Long March 5 rocket from Xichang, China.
o The scientific goals of the mission include studying Martian topography and geology and
determining the composition of the surface material, climate and environment.
o Tianwen-1 will reach the Red Planet’s orbit in February 2021. The rover will land on Mars in May.
• So far, US, Russia, India and European Union have successfully sent missions to Mars. India was
the first Asian country to send Mars orbiter and first country anywhere to do so on its first attempt.

Q 16.B
• The initial demands of the nationalist Indians were rather mild, to avoid any confrontation with the British
rule. And when denied it also awakened the Indian people about the hegemony of the colonial rule and not
granting basic rights and demands to the people.
• The nationalist Indian demands of those years included -
o Indianization of the civil services. Hence option 1 is correct.
o no reduction of import duties on textile imports, as this would make foreign goods cheaper
compared to local and put Indian handicrafts and artisans into misery. Hence option 2 is not correct.
o no expansion in Afghanistan or Burma
o the right to bear arms. Hence option 3 is correct.
o freedom of the press
o reduction of military expenditure
o higher expenditure on famine relief
o the right of Indians to join the semi-military volunteer corps
o the right of Indian judges to try Europeans in criminal cases
o the appeal to British voters to vote for a party which would listen to Indians.

Q 17.C
• The background to the Khilafat movement was provided by a series of events after the First World War.
The Treaty of Sevres with Turkey, signed in May 1920, completely dismembered Turkey and sanctions
imposed upon the Caliphate, who was considered as supreme leader of muslims all over the world.
• Indian Muslims were critical of the treatment meted out to Turkey (Ottoman Empire) under Treaty of
Sevres. Khilafat movement was launched to influence British in its treatment of the Ottoman Empire post
World War 1.
• Mohammad Ali and his brother Maulana Shaukat Ali joined with other Muslim leaders to form the
All India Khilafat Committee which called upon the British to protect the Caliphate and for Indian
Muslims to unite and hold the British accountable for this purpose.
• Khilfat Leaders and Indian national congress launched Khilafat movement to influence Britishers
in its treatment of the Ottoman Empire post World War 1 and pursue the goals of Indian National
movement as well. Hence option (c) is the correct answer.

6 www.visionias.in ©Vision IAS

Google it:- https://upscpdf.com


https://t.me/UPSC_PDF Download From > https://upscpdf.com https://t.me/UPSC_PDF

Q 18.C
• A radical trend arose among the Bengali intellectuals during the late 1820's and 1830's. It was more
modern than even Rammohan Roy's and is known as Young Bengal Movement founded by Henry
Vivian Derozio. He followed the most radical views of the time drawing his inspiration from the great
French Revolution. Hence option (c) is the correct answer.
• He inspired his students and followers the Derozians and Young Bengal, to think rationally and freely, to
question all authority, to love libery equality and freedom, and to worship truth.
• The Derozians attacked old and decadent customs, rites and traditions. They were passionate advocates of
women's rights and demanded education for them.
• However, the social conditions then were not ripe for their ideas to flourish, they did not take up the
peasant's cause and no other class supported their advanced ideas.

Q 19.D
• Bal Gangadhar Tilak was an ardent nationalist who helped lay the foundation for India’s independence by
building his own defiance of British rule into a national movement.
• He was most frequently associated with the struggle for the freedom of the Press during the nationalist
movement. In 1881, along with G.G. Agarkar, he founded the newspaper Kesari (in Marathi) and
Mahratta (in English).
• In 1893, he started the practice of using the traditional religious Ganapati festival to propagate
nationalist ideas through patriotic songs and speeches. In 1896, he started the Shivaji festival to
stimulate nationalism among young Maharashtrians. In the same year, he organized an all-Maharashtra
campaign for the boycott of foreign cloth in protest against the imposition of the excise duty on cotton.
• He was, perhaps the first among the national leaders to grasp the important role that the lower middle
classes, peasants, artisans and workers could play in the national movement and, therefore, he saw the
necessity of bringing them into the Congress fold.
• Tilak started his Home League in April 1916 and covered the area of Maharashtra (excluding Bombay
city), Karnataka, Central Province and Berar.
• In 1916 he concluded the Lucknow Pact with Mohammed Ali Jinnah, which provided for Hindu-
Muslim unity in the nationalist struggle.
• Hence all the statements are correct.

Q 20.D
• India and the US have long-standing energy collaboration. In June 2017, the Indian Prime Minister and
US President reinforced the strategic importance of bilateral energy cooperation, through the
announcement of a new US–India Strategic Energy Partnership (SEP). The first ministerial meeting was
held in April 2018.
• The India–US Strategic Energy Partnership has four pillars: Oil and Gas, Power and Energy Efficiency,
Renewable and Sustainable Growth. The Sustainable Growth pillar is being co-chaired by NITI Aayog
and USAID.
• Key achievements and future action plan of the Sustainable Growth pillar were highlighted during the
ministerial dialogue held recently:
o Energy data management: An India Energy Dashboard, which was launched in 2015, has been
revamped with the provisioning of data input online and through API integration.
o Energy modeling: Two exercises carried out on energy-water nexus and decarburization of the
transportation sector have highlighted key issues and provided policy advice. NITI Aayog and
USAID jointly launched India Energy Modeling Forum on 2 July 2020. The forum will engage
US and Indian researchers, knowledge partners, think tanks, both national and international, and
government agencies and departments for modeling and long-term energy planning exercise. Hence,
option (d) is the correct answer.
o Low carbon technologies: Both sides agreed to engage through concerned Government, related
agencies, and private players for the promotion of low carbon technologies in India.

Q 21.D
• The Bardoli Satyagraha movement sparked off in 1926 when the authorities decided to increase the
land revenue by 30 per cent. The Congress leaders set up a Bardoli Inquiry Committee to go into the
issue. The committee found the revenue hike to be unjustified. Vallabhbhai Patel was called to lead
the movement for the withdrawal of the enhanced land revenue. Hence option (d) is the correct
answer.

7 www.visionias.in ©Vision IAS

Google it:- https://upscpdf.com


https://t.me/UPSC_PDF Download From > https://upscpdf.com https://t.me/UPSC_PDF

• Under Patel, the Bardoli peasants resolved to refuse payments of the revised assessment until the
Government appointed an independent tribunal or accepted the current amount as full payment. To
organise the movement, Patel set up 13 chhavanis or workers’ camps in the taluqa. The women of
Bardoli gave him the title of 'Sardar'.
• Those who opposed the movement faced a social boycott. Special emphasis was placed on the
mobilisation of women. K.M. Munshi and Lalji Naranji resigned from the Bombay Legislative Council
in support of the movement.
• By August 1928, massive tension had built up in the area. There were prospects of a railway strike in
Bombay. Gandhi reached Bardoli to stand by in case of any emergency. The Government appointed a
committee that went into the whole affair and found the revenue hike to be unjustified and
recommended a rise of 6.03 per cent only.

Q 22.C
• In an endeavor to improve the information flow and bridge the demand-supply gap in the skilled
workforce market, the Ministry of Skill Development and Entrepreneurship (MSDE) recently
launched ‘Aatamanirbhar Skilled Employee Employer Mapping (ASEEM)’ portal to help skilled
people find sustainable livelihood opportunities. Hence, option (c) is the correct answer.
• Apart from recruiting a skilled workforce that spurs business competitiveness and economic growth, the
Artificial Intelligence-based platform has been envisioned to strengthen their career pathways by
handholding them through their journeys to attain industry-relevant skills and explore emerging job
opportunities especially in the post COVID era. The portal will map details of workers based on regions
and local industry demands.
• Database of labour migrants in Indian states and overseas citizens, who returned to India under the Vande
Bharat Mission and filled SWADES Skill Card, has been integrated with the ASEEM portal.
• The portal consists of three IT based interfaces:
o Employer Portal – Employer onboarding, Demand Aggregation, candidate selection
o Dashboard – Reports, Trends, analytics, and highlight gaps
o Candidate Application – Create & Track candidate profile, share job suggestion

Q 23.C
• Lord Cornwallis realized the need for police reforms and he made many changes in the police
organization.
• In this respect , he went back to , and modernised the , old Indian system of thanas. This put India ahead
of Britain where a system of police had not developed yet.
• He divested the Zamindars of their policing powers, divided the district into thanas or units of
police jurisdiction of twenty to thirty miles. Hence statement 2 is correct.
• Each unit was under an officer known as the Daroga who was appointed by the magistrates and placed
under their supervision. Darogas came to be seen as the instrument of the Company's power and control
over the rural areas. The daroga was an Indian. Hence statement 1 is not correct.
• In the villages, the duties of the police continued to be performed by the village watchmen who were
maintained by the villagers. Hence statement 3 is correct.
• The police gradually succeeded in reducing major crimes like dacoity.

Q 24.B
• Since the Moon is tidally locked with Earth, we only see one side of the Moon, which is also known as the
‘near side’. This Earth-facing side is perpetually different from the ‘far side’ that always faces away from
Earth. A recent research paper proposed an explanation for this asymmetry. As per the paper,
radioactive decay of certain elements like Uranium and Thorium in lunar maria may be responsible for
imminent asymmetry of the Moon. The near side features dark regions called ‘lunar maria’ which are
believed to be craters or volcanic features. Hence option (b) is the correct answer.
• The lunar maria are large, dark, basaltic plains on Earth's Moon, formed by ancient volcanic
eruptions. They were dubbed maria, Latin for "seas", by early astronomers who mistook them for actual
seas.
• The maria cover about 16% of the lunar surface, mostly on the side visible from Earth. The few maria on
the far side are much smaller, residing mostly in very large craters. There is a very, very thin layer of
gases on the lunar surface that can almost be called an atmosphere. Technically, it's considered an
exosphere.

8 www.visionias.in ©Vision IAS

Google it:- https://upscpdf.com


https://t.me/UPSC_PDF Download From > https://upscpdf.com https://t.me/UPSC_PDF

• One of the sources for the moon's atmosphere is outgassing, the release of gases from the lunar interior,
usually due to radioactive decay. Outgassing events may also occur during moonquakes. After being
released, lighter gases escape into space almost immediately. Outgassing replenishes the tenuous
atmosphere.
• A 22° halo is an optical phenomenon that belongs to the family of ice-crystal halos. Its form is a ring
with an apparent radius of approximately 22° around the Sun or Moon. When visible around the Moon, it
is called a moon ring or winter halo. It forms as direct sunlight or moonlight is refracted in millions of
hexagonal ice crystals suspended in the atmosphere. The halo appears large; its radius is roughly the
length of an outstretched hand at arm's length. A 22° halo may be visible on as many as 100 days per year
– much more frequently than rainbows
• A transient lunar phenomenon (TLP) or lunar transient phenomenon (LTP) is a short-lived light, color
or change in appearance on the surface of the Moon. The term was coined in one of the reports of NASA.
The transient phenomenon may be a result of events such as outgassing.

Q 25.C
• The historic Dandi march, marking the launch of the Civil Disobedience Movement, began on March
12, 1930, and Gandhi broke the salt law by picking up a lump of salt at Dandi (Gujarat) on April
6. The violation of the law was seen as a symbol of the Indian people’s resolve not to live under British-
made laws and therefore under British rule. Gandhi openly asked the people to make salt from sea water
in their homes and violate the salt law. Hence statement 1 is correct.
o Once the way was cleared by Gandhi’s ritual at Dandi, defiance of the salt laws started all over the
country.
o In April 1930, C. Rajagopalachari organised a march from Thiruchirapalli (Trichinapoly as it
was called by the British) to Vedaranniyam on the Tanjore coast to break the salt law. Hence
statement 2 is not correct.
o In Malabar, K. Kelappan, the hero of the Vaikom Satyagraha, walked from Calicut to Payannur to
break the salt law.
o In the Andhra region, a number of sibirams (military-style camps) were set up to serve as the
headquarters of the Salt Satyagraha.
• On May 21, with Sarojini Naidu, the first Indian woman to become President of the Congress, and Imam
Saheb, Gandhiji’s comrade of the South African struggle, at the helm, and Gandhiji’s son, Manilal, in
front ranks, a band of 2000 marched towards the police cordon that had sealed off the Dharasana salt
works. The unarmed and peaceful crowd was met with a brutal lathi charge which left 2 dead and 320
injured. Hence statement 3 is correct.

Q 26.C
• Hagia Sophia is a 1,500-year-old structure in Turkey. Listed as a UNESCO World Heritage site, it was
originally a cathedral before it was recently turned into a mosque. In the 1930s, Mustafa Kemal Ataturk,
the founder of the Republic of Turkey, shut down the mosque and turned it into a museum in an attempt to
make the country more secular. A government decree reopened the "jewel" of the Byzantine Empire for
Muslim worship and abolished its status as a museum. The conversion of what was once the most
important church of Christendom has led to an international outcry. Hence, pair 1 is correctly matched.
• Bhashan Char is an uninhabited island where the government of Bangladesh has made shelter houses
for one lakh Rohingya refugees. Bangladesh last year constructed facilities for 100,000 people on
Bhashan Char, a silt islet in the coastal belt, saying they needed to take the pressure off crowded border
camps that are home to almost one million Rohingya. Hence, pair 2 is not correctly matched.
• In a bid to boost its economic and connectivity linkages with its neighbors, India in recent days has
opened a new route for trade with its key neighbor Bhutan. The new trade route was opened through
Ahllay near Pasakha in south-eastern Bhutan. Hence, pair 3 is correctly matched.

Q 27.A
• Individual Satyagraha: In 1940, then Viceroy Linlithgow made a set of proposals called the ‘August
offer’ which included that a representative Indian body would frame a constitution for India after the war
and Dominion status is the objective for India. Indian National Congress rejected this offer in its meeting
at Wardha in August 1940 and demanded complete freedom from the colonial rule. Jawaharlal Nehru
remarked that the dominion status concept was as dead as a doornail. After this, Mahatma Gandhi
initiated the Individual Satyagraha to affirm the right to free speech. He avoided a mass satyagraha
because he did not want violence.

9 www.visionias.in ©Vision IAS

Google it:- https://upscpdf.com


https://t.me/UPSC_PDF Download From > https://upscpdf.com https://t.me/UPSC_PDF

• The aims/methodology of Individual Satyagrahis were:


o To show that nationalist patience was not due to weakness.
o To express people’s feelings that they were not interested in the war and that they made no distinction
between Nazism and the double autocracy that ruled India.
o To give another opportunity to the Government to accept Congress’s demands peacefully.
o The demand of the Satyagrahi was using Freedom of Speech against the war through an anti-war
declaration. If the government did not arrest the Satyagrahi, he or she will move repeating it in
villages and start march towards Delhi (“Delhi Chalo Movement”). The centerpiece of
Individual Satyagraha was non-violence which could be achieved only by selecting the
Satyagrahis.
• Acharya Vinoba Bhave, Pt. Jawaharlal Nehru, and Brahma Dutt were the first, second, and third
the selected Satyagarhi respectively. Hence option (a) is correct. Till 15 May 1941, 25000
satyagrahis had been jailed.
• Thus, the Individual Satyagraha had a dual purpose - while giving expression to the Indian people’s strong
political feeling, it gave the British government further opportunity to peacefully accept the Indian
demands. Gandhiji and the Congress were, because of their anti-Nazi feelings, still reluctant to take
advantage of’ the British predicament and embarrass her war effort by a mass upheaval in India. More
importantly, Gandhiji was beginning to prepare the people for the coming struggle. The Congress
organization was being put back in shape; opportunist elements were being discovered and pushed out of
the organization; and above all the people were being politically aroused, educated, and mobilized.

Q 28.A
• The Quadrilateral Security Dialogue (QSD, also known as the Quad) is an informal strategic forum
between the United States of America, Japan, Australia and India. Hence the correct answer is
option (a).
o It is maintained by semi-regular summits, information exchanges and military drills between member
countries, Exercise Malabar being the major one.
o It was initiated as a dialogue in 2007 by Prime Minister Shinzo Abe of Japan, with the support of
leaders of the other three countries.
o It was widely viewed as a response to increased Chinese economic and military power especially its
maneuvers in the South China Sea and the Indian Ocean.
o Quad temporarily ceased to exist following the withdrawal of Australia in February 2008 due to
diplomatic protests from China.
o However, negotiations to revive the quadrilateral alliance were held during the 2017 ASEAN
Summits.
o Recently, US supercarriers carried out a joint exercise with Indian Navy in the Indian Ocean,
and a second one with the other two QUAD members - Australia and Japan - in the Philippine
Sea. Twin naval exercises with US supercarriers signal towards the complete revival of QUAD.
o The four navies are expected to participate in the Malabar naval exercise slated to be conducted in
November.

Q 29.B
• The nationalist agitation forced the Government to make some changes in legislative functioning by the
Indian Councils Act of 1892. The number of additional members of the Imperial and Provincial
Legislative Councils was increased from the previous six to ten to ten to sixteen. A few of these members
could be elected indirectly through municipal committees, district boards, etc., but the official majority
remained. The members were given the right to discuss the annual budget but they could neither vote on it
nor move a motion to amend it. They could also ask questions but were not allowed to put supplementary
questions or to discuss the answers.
• The ‘reformed’ Imperial Legislative Council met, during its tenure till 1909, on an average for only
thirteen days in a year, and the number of unofficial Indian members present was only five out of twenty-
four.
• The nationalists were totally dissatisfied with the Act of 1892. They saw in it a mockery of their
demands. The Councils were still impotent; despotism still ruled. They now demanded a majority for
non-official elected members with the right to vote on the budget and, thus, to the public purse. They
raised the slogan ‘no taxation without representation.’ Gradually, they raised their demands. Hence,
option (b) is correct.

10 www.visionias.in ©Vision IAS

Google it:- https://upscpdf.com


https://t.me/UPSC_PDF Download From > https://upscpdf.com https://t.me/UPSC_PDF

Q 30.A
• The Union HRD Ministry recently released PRAGYATA Guidelines on Digital Education. Hence,
option (a) is correct.
• The PRAGYATA guidelines include eight steps of online education that is, Plan, Review, Arrange,
Guide, Yak(talk), Assign, Track, and Appreciate
• COVID-19 pandemic has led to the closure of schools and has impacted over 240 million children of the
country who are enrolled in schools. Extended school closures may cause loss of learning. To mitigate the
impact of the pandemic, schools will not only have to remodel and reimagine the way teaching and
learning have happened so far but will also need to introduce a suitable method of delivering quality
education through a healthy mix of schooling at home and schooling at school.
• PRAGYATA guidelines have been developed from the perspective of learners, with a focus on
online/blended/digital education for students who are presently at home due to lockdown. The guidelines
will be relevant and useful for a diverse set of stakeholders including school heads, teachers, parents,
teacher educators and students.
• The guidelines stress upon the use of an alternative academic calendar of NCERT, for both, learners
having access to digital devices and learners having limited or no access.
• Guidelines recommend a cap on duration and number of online sessions in a day for students from Class I
to XII.

Q 31.B
• Statement 1 is not correct: In 1936, Lucknow Session of Congress, Swami Sahajanand Saraswati
founded All-India Kisan Congress which was later changed to All-India Kisan Sabha. Swami
Sahajanand, the militant founder of the Bihar Provincial Kisan Sabha (1929), was elected the
President and N.G. Ranga, the pioneer of the Kisan movement in Andhra and a renowned scholar
of the agrarian problem, the General Secretary.
• The first session in Bombay was greeted in person by Jawaharlal Nehru. Other participants included
Ram Manohar Lohia, Sohan Singh Josh, Indulal Yagnik, Jayaprakash Narayan, Mohanlal Gautam, Kamal
Sarkar, Sudhin Pramanik and Ahmed Din. The Sabha resolved to bring out a Kisan Manifesto and a
periodic bulletin edited by Indulal Yagnik.
• A Kisan Manifesto which was finalized at the All-India Kisan Sabha session in Bombay was formally
presented to the Congress Working Committee to be incorporated into its forthcoming manifesto for the
1937 elections. The Kisan Manifesto considerably influenced the agrarian program adopted by the
Congress at its Faizpur session, which included demands for fifty per cent reduction in the land revenue
and rent, a moratorium on debts, the abolition of feudal levies, the security of tenure for tenants, a living
wage for agricultural laborers, and the recognition of peasant unions.
• Statement 2 is correct: At Faizpur, in Maharashtra, along with the Congress session under the
Presidentship of Jawahar Lal Nehru, was held the second session of the All India Kisan Sabha
(presided over by N.G. Ranga). Five hundred kisans marched for over 200 miles from Manmad to
Faizpur educating the people along the way about the objects of the Kisan Congress. They were
welcomed at Faizpur by Jawaharlal Nehru, Shankar Rao Deo, M.N. Roy, Narendra Dev, S.A. Dange,
M.R. Masani, Yusuf Meherally, Bankim Mukherji and many other Kisan and Congress leaders. This was
the first session of the Congress to be held in a village due to the influence of AIKS.

Q 32.A
• Accelerate Vigyan Scheme was recently launched by Department of Science and Technology’s (DST’s)
Science and Engineering Research Board (SERB). It is under the Ministry of Science and Technology. It
is an Inter-Ministerial Initiative. Hence, statement 2 is not correct.
• The scheme was launched with the objective to provide a single platform that will encourage high-end
scientific research in the country by providing a common forum for conducting workshops, internships
and capacity building programs for research scholars and students. Hence, statement 1 is correct.
• The Accelerate Vigyan scheme begins with the Mission SAMOOHAN (Bring Together). This mission is
for consolidating all scientific interactions in the country under one common roof. This mission has two
components: SANGOSHTHI (Seminar & Symposia) & SAYONJIA (Chronicle).
• ABHYAAS, a program of AV scheme, is an attempt to boost Research & Development in the country by
enabling and grooming potential PG and Ph.D. level students by developing dedicated research skills in
selected areas/disciplines/fields through its two components- High-End Workshops 'KARYASHALA’ and
Research Internships ‘VRITIKA.’ This will be especially important for those researchers who have
limited opportunities to access such learning capacities/ facilities/infrastructure.

11 www.visionias.in ©Vision IAS

Google it:- https://upscpdf.com


https://t.me/UPSC_PDF Download From > https://upscpdf.com https://t.me/UPSC_PDF

• SERB also launched another new component, ‘SAYONJIKA’ under AV, as an open-ended program on 01
July 2020 to catalogue the capacity building activities in S&T supported by all government funding
agencies in the country.

Q 33.A
• Statement 1 is correct: Frightened by the spread of socialist and communist ideas and influence and
believing that the crucial role in this respect was being played by British and other foreign agitators sent to
India by the Communist International, the Government proposed to acquire the power to deport
‘undesirable’ and ‘subversive’ foreigners. This was done by introducing Public Safety Bill, 1928 in the
Legislative Council.
• Statement 2 is not correct: Nationalists of all colors, from the moderates to the militants, united in
opposing the Bill. Lala Lajpat Rai said, ‘Capitalism is only another name for Imperialism... We are in no
danger from Bolshevism or Communism. The greatest danger we are in is from the capitalists and
exploiters.'' Motilal Nehru(Swarajist) narrated his experiences in the Soviet Union and condemned
anti-Soviet propaganda. He described the Public Safety Bill as ‘a direct attack on Indian
nationalism, on the Indian National Congress’ and as ‘the Slavery of India, Bill No. 1.’ T. Prakasam
said that Bill’s main aim was to prevent the spread of nationalism among workers and peasants.’ Diwan
Chaman Lall, then a firebrand protege of Motilal, declared: ‘If you are trying to preach against socialism
if you are demanding powers to suppress socialism, you will have to walk over our dead bodies before
you can get that power.’ Even the two spokesmen of the capitalist class, Purshottamdas Thakurdas,
and G.D. Birla, firmly opposed the Bill. The Bill was defeated in the Legislative Council as all the
Indians opposed the Bill including Liberals like M.R. Jayakar.
• In March 1929, having failed to get the Bill passed, the Government arrested thirty-one leading
communists, trade unionists, and other leftwing leaders and put them on trial at Meerut. This led to
strong criticism of the Government by the nationalists.

Q 34.B
• In the nineteenth century, the western-educated intellectuals like Dadabhai Naoroji (Drain Theory),
Mahadev Govind Ranade, R.C. Dutt (The Economic History of India), G.V. Joshi, G. Subramaniya Iyer,
G.K Gokhale, etc. clearly understood the predatory economic policies of the British and became stern
critics.
• For their entire lives, they carried on national awakening among Indian masses on economic grounds.
• G.K. Gokhale: He was a senior leader of the Indian National Congress and the founder of the Servants of
India Society. He was trained in Indian economics by Justice Ranade and G.V. Joshi. He gained great
fame for his budget speeches. Mahatma Gandhi called Gokhale his mentor and guide.
• G.V. Joshi: popularly known as Sarwajanik Kaka, was a lawyer, social reformer, and political activist. He
was a founding member of Poona Sarvajanik Sabha. G.V. Joshi along with Ranade created the initial
public opinion among the intelligentsia on problems of political economy and issues of economic
development.
• Syed Ahmed Khan: He was more of an Islamic pragmatist, Islamic reformer, and philosopher of
nineteenth-century British India. In 1859, Syed established Gulshan School at Muradabad, Victoria
School at Ghazipur in 1863, and a scientific society for Muslims in 1864. In 1875, founded the
Muhammadan Anglo-Oriental College, the first Muslim university in Southern Asia. During his career,
Syed repeatedly called upon Muslims to loyally serve the British Raj and criticized the Indian National
Congress. Hence option (b) is the correct answer.

Q 35.C
• Under Gandhi-Irwin pact, Mahatma Gandhi on behalf of Congress agreed to participate in the second
Round Table Conference on the constitutional question. The second Round Table Conference was held
in London from September 7, 1931 to December 1, 1931. The Indian National Congress nominated
Gandhi as its sole representative.
• On the failure of the second Round Table Conference, the Congress Working Committee decided on
December 29, 1931 to resume the civil disobedience movement. This phase of the movement could not be
sustained for long because of
o Severe Government repression
o Gandhi and other leaders had no time to build up the tempo; and
o the masses were not prepared.
o Finally in April 1934, Gandhi decided to withdraw the civil disobedience movement.

12 www.visionias.in ©Vision IAS

Google it:- https://upscpdf.com


https://t.me/UPSC_PDF Download From > https://upscpdf.com https://t.me/UPSC_PDF

• The All India Students' Federation (AISF) is the oldest student organisation in India. It was founded on
12 August 1936 in Lucknow. The foundation of the conference was one of the other major achievements
of the Left.
• Jawaharlal Nehru was nominated the president for the Lahore session of the Congress (December 1929)
mainly due to Gandhi’s backing. In the session, January 26, 1930 was fixed as the first Independence
(Swarajya) Day, to be celebrated everywhere. On this day, Public meetings were organised all over the
country in villages and towns and the independence pledge was read out in local languages and the
national flag was hoisted.
• Hence the correct order is 3-1-2.

Q 36.D
• The word Mahalwari is derived from the Hindi word Mahal, which means house, district, neighborhood or
quarter. This system consisted of landlords or lambardars claiming to represent entire villages or even
groups of villages. Along with the village communities, the landlords were jointly responsible for the
payment of the revenues. But, there was individual responsibility. The land included under this system
consisted of all land of the villages, even the forestland, pastures, etc.
• The settlement under the Mahalwari System was made directly with the villages or estates or Mahals by
the instructions of the settlement officers, who fixed the rent with the consultation of lambardars. The
assessment was done every 30 years and in some areas every 20 years.
• The Mahalwari system was introduced by Francis Hastings & R.M. Bird in 1822. The other two
systems were the Permanent Settlement of Bengal in 1793 and the Ryotwari system in 1792.
• It covered the States of Punjab, Awadh, and Agra, parts of Orissa and Madhya Pradesh.
• It was introduced in NWFP in 1822 by Regulation VIII in 1822. This Regulation was introduced by Holt
Mackenzie.
• It was further extended from Central Provinces to British Punjab.
• The settlement under the Mahalwari System was made directly with the villages or estates
or Mahals by the instructions of the settlement officers, who fixed the rent with the consultation
of lambardars.
• The assessment was done every 30 years and in some areas every 20 years.
• The word "Mahalwari" is derived from the Hindi word Mahal, which means house, district, neighborhood
or quarter. This system consisted of landlords or lambardars claiming to represent entire villages or even
groups of villages. Along with the village communities, the landlords were jointly responsible for the
payment of the revenues. But, there was individual responsibility. The land included under this system
consisted of all land of the villages, even the forestland, pastures etc.

Q 37.A
• There were multiple storm centres of 1857 revolt which were led by various leaders.
• At Delhi, the nominal and symbolic leadership belonged to the Mughal emperor, Bahadur Shah, but the
real command lay with a court of soldiersheaded by General Bakht Khan.
• At Kanpur, the natural choice was Nana Saheb, the adopted son of the last Peshwa, Baji Rao II. He
was refused the family title and banished from Poona, and was living near Kanpur. Hence pair 1 is
correctly matched.
• Begum Hazrat Mahal took over the reigns at Lucknow, where the rebellion broke out on June 4, 1857
and popular sympathy was overwhelmingly in favour of the deposed nawab. Her son, Birjis Qadir, was
proclaimed the nawab and a regular administration was organised with important offices shared equally
by Muslims and Hindus. Hence pair 4 is correctly matched.
• At Bareilly, Khan Bahadur, a descendant of the former ruler of Rohilkhand was placed in
command. Not enthusiastic about the pension being granted by the British, he organized an army of
40,000 soldiers and offered stiff resistance to the British. Hence pair 2 is not correctly matched.
• In Bihar, the revolt was led by Kunwar Singh, the zamindar of Jagdishpur. He unhesitatingly joined
the sepoys when they reached Arrah from Dinapore (Danapur). Hence pair 3 is correctly matched.
• Maulvi Ahmadullah of Faizabad fought a stiff battle against the British troops. He emerged as one of the
revolt’s acknowledged leaders once it broke out in Awadh in May 1857.
• Rani Laxmibai, assumed the leadership of the sepoys at Jhansi. The Rani of Jhansi and Tantia Tope
marched towards Gwalior. Gwalior was recaptured by the English in June 1858.

13 www.visionias.in ©Vision IAS

Google it:- https://upscpdf.com


https://t.me/UPSC_PDF Download From > https://upscpdf.com https://t.me/UPSC_PDF

Q 38.C
• The United Nations Environment Programme (UNEP) and the International Energy Agency
(IEA) have jointly released a report titled ‘Cooling Emissions and Policy Synthesis Report: Benefits of
cooling efficiency and the Kigali Amendment’ report.
• It is based on the assessment of development and climate benefits of efficient and climate-friendly
cooling.
• Key Findings:
o Increased cooling demand is contributing to the emissions of HFC, CO2 and black carbon.
o Direct and indirect emissions are projected to rise by 90% above 2017 levels.
o The world will need more than 10 billion new cooling appliances by 2050, which will make the count
of such machines to 14 billion.
o Around 3.6 billion appliances are being used and it is increasing by up to 10 cooling appliances every
second.

Q 39.D
• The union Ministry of Human Resource Development recently launched MANODARPAN initiative to
provide psychosocial support to students for their mental health and well-being during the COVID
outbreak and beyond. Hence, option (d) is correct.
• The following components are included in the MANODARPAN initiative:
o Advisory Guidelines for students, teachers and faculty of School systems and Universities along with
families.
o Web page on the MHRD website, which will carry advisory, practical tips, posters, videos, do’s and
don’ts for psychosocial support, FAQs and online query system.
o National level database and directory of counsellors at School and University level whose services
can be offered voluntarily for Tele-Counselling Service on the National Helpline.
o National Toll-free Helpline by the MHRD for a country wide outreach to students from school,
universities and colleges. This unique helpline shall be manned by a pool of experienced counselors/
Psychologists and other mental health professionals and will continue beyond the COVID-19
situation.
o Handbook on Psychosocial Support: Enriching Life skills & Wellbeing of Students’ to be published
online. The booklet will include FAQs, Facts & Myths and will also cover ways and means to manage
emotional and behavioural concerns (from young children to college youth) during the COVID-19
pandemic and beyond.
o Interactive Online Chat Platform for contact, counselling and guidance by psychologists and other
mental health professionals which will be available for students, teachers, and families during
COVID-19 and beyond.
o Webinars, audio-visual resources including videos, posters, flyers, comics, and short films to be
uploaded as additional resource materials on the webpage. Crowd sourcing from students all over the
country will be encouraged as peer support.

Q 40.B
• The Scientific Society of Aligarh or The Scientific Society was founded by Sir Syed Ahmed Khan in
1864. In 1862 Syed formed a Translation Society which used to translate the scientific books of English
and other European languages into Urdu and Hindi. This society later evolved into the Scientific Society
of Aligarh. The society sought to promote liberal, modern education and Western scientific knowledge in
the Muslim community in India. Hence pair 1 is correctly matched.
• In 1866, Dadabhai Naoroji organised the East India Association in London to discuss the Indian
question and to influence British public men to promote Indian welfare. Later he organized branches
of the association in prominent Indian cities. Hence pair 2 is correctly matched.
• In May 1884., S.Ramaswami Mudaliar, P. Rangaiah Naidu, P. Anandacharlu and G. Subramaniya
Iyer established the Madras Mahajana Sabha. The Sabha adopted a moderate policy in its early days.
However, still, its aims and objectives were considered seditious. In December 1895, on his visit to
Madras, the Viceroy of India, Lord Elgin refused to receive the welcome address from the Madras
Mahajana Sabha.
• Subramaniya Siva was an Indian freedom fighter, writer and pure Tamil movement activist, born in
1884. Hence pair 3 is not correctly matched.

14 www.visionias.in ©Vision IAS

Google it:- https://upscpdf.com


https://t.me/UPSC_PDF Download From > https://upscpdf.com https://t.me/UPSC_PDF

Q 41.A
• In the 1930s, in the Malabar region of Kerala, a powerful peasant movement developed as the result of the
efforts mainly of Congress Socialist Party activists. They toured villages and set up Karshak
Sanghams (peasant associations). Hence option (a) is the correct answer.
• The main forms of mobilization and agitation were
o the formation of village units of the Karshaka Sanghams,
o conferences and meetings.
o the marching of jathas or large groups of peasants to the houses of big jenmies or landlords, placing
the demands before them and securing immediate redressal.
o The main demand of these jathas was for the abolition of feudal levies such as vasi, nuri, etc.
• The Karshaka Sanghams also organized a powerful campaign around the demand for amending
the Malabar Tenancy Act of 1929.
• A committee headed by R. Ramachandra Nedumgadi was appointed by the All Malabar Karshaka
Sangham to enquire into the tenurial problem, and its recommendations were endorsed by the Kerala
Pradesh Congress Committee in November 1938.

Q 42.C
• The main form of political work by the early Communists was to organize peasants’ and workers’
parties and work through them. The first such organization was the Labour-Swaraj Party of the
Indian National Congress organized by Muzaffar Ahmed, Qazi Nazrul Islam, Hemanta Kumar
Sarkar, and others in Bengal in November 1925. Hence option (c) is the correct answer.
• In late 1926, a Congress Labour Party was formed in Bombay and a Kirti-Kisan Party in Punjab. A
Labour Kisan Party of Hindustan had been functioning in Madras since 1923. By 1928 all of these
provincial organizations had been renamed the Workers’ and Peasants’ Party (WPP) and knit into
an All India party, whose units were also set up in Rajasthan, UP, and Delhi.
• All Communists were members of this party. The basic objective of the WPPs was to work within the
Congress to give it a more radical orientation and make it ‘the party of the people’ and independently
organize workers and peasants in class organizations, to enable first the achievement of complete
independence and ultimately of socialism. The WPPs grew rapidly and within a short period, the
communist influence in Congress began to grow rapidly, especially in Bombay. Moreover, Jawaharlal
Nehru and other radical Congressmen welcomed the WPPs’ efforts to radicalize the Congress. Along with
Jawaharlal and Subhas Bose, the youth leagues and other Left forces, the WPPs played an important role
in creating a strong left-wing within the Congress and in giving the Indian national movement a leftward
direction.
• The WPPs also made rapid progress on the trade union front and played a decisive role in the resurgence
of working class struggles during 1927-29 as also in enabling in Communists to gain a strong position in
the working class.
• Later WPP merged in the Communist Party of India in the 1930s.

Q 43.B
• The Government of India Act 1858 was an Act of the Parliament which was originally titled as An
Act for the Better Government of India. Hence statement 1 is correct.
• Under the Act, the government was to be carried on as before by the Governor-General who was also
given the title of viceroys or crowns personal representatives. Hence statement 2 is not correct.
• An Act of the Parliament in 1858 transferred the power to govern from the East India Company to the
British Crown.
• While authority over India has previously been wielded by the Directors of the Company and the Board of
Control, now this power was to be exercised by a Secretary of State for India aided by the council.
• Secretary of State was a member of the British Cabinet and as such responsible to Parliament.
Hence statement 3 is correct

Q 44.C
• The Consumer Protection Act, 2019 came into force recently, on 20th July 2020.
• Under the Act, a consumer is defined as a person who buys any good or avails a service for a
consideration. It does not include a person who obtains a good for resale or a good or service for
commercial purpose. It covers transactions through all modes including offline, and online through
electronic means, teleshopping, multi-level marketing or direct selling. Hence, statement 1 is not
correct.

15 www.visionias.in ©Vision IAS

Google it:- https://upscpdf.com


https://t.me/UPSC_PDF Download From > https://upscpdf.com https://t.me/UPSC_PDF

• The Act provides for product liability. Product liability means the liability of a product manufacturer,
service provider or seller to compensate a consumer for any harm or injury caused by a defective good or
deficient service. To claim compensation, a consumer has to prove any one of the conditions for defect or
deficiency, as given in the Act. Hence, statement 2 is correct.
• An Alternate Dispute Resolution mechanism of Mediation has been provided in the Act, which
simplifies the adjudication process. A complaint will be referred by a Consumer Commission for
mediation, wherever scope for early settlement exists and parties agree for it. Mediation will be held in the
Mediation Cells to be established under the aegis of the Consumer Commissions. There will be no appeal
against settlement through mediation. Hence, statement 3 is not correct.

Q 45.A
• Vaccine-derived polioviruses (VDPVs) are rare strains of poliovirus that have genetically mutated from
the strain contained in the oral polio vaccine. As per recent stats released by WHO, around 150 VDPD
cases have been reported in the last 60 years. Hence, statement 1 is correct.
• The oral polio vaccine contains a live, attenuated(weakened) vaccine-virus. When a child is vaccinated,
the weakened vaccine-virus replicates in the intestine and enters into the bloodstream, triggering a
protective immune response in the child. Like wild poliovirus, the child excretes the vaccine-virus for a
period of six to eight weeks. Importantly, as it is excreted, some of the vaccine-virus may no longer be the
same as the original vaccine-virus as it has genetically altered during replication. This is called a VDPV.
• Vaccine-derived polioviruses must be managed in the same way as wild poliovirus outbreaks. The
solution is the same for all polio outbreaks: vaccinate every child several times with oral polio vaccine to
stop polio transmission, regardless of whether the virus is wild or vaccine-derived. Outbreaks are usually
rapidly stopped with 2–3 rounds of high-quality supplementary immunization activities. Hence,
statement 3 is not correct.
• There are total of three wild poliovirus strains.
• Type 2 and Type 3 (WPV3) has been eradicated worldwide and Type 1 is yet to be eradicated.
• Once wild poliovirus transmission has been stopped globally, the vaccine-viruses will be the only source
of live polioviruses in the community and could potentially lead to the re-emergence of polio. Use of the
oral polio vaccine in routine immunization programmes will therefore be phased out to eliminate the rare
risks posed by vaccine-derived polioviruses. Hence, statement 2 is not correct.

Q 46.C
• On 4 September 1920, Congress met at Calcutta in a special session. This special session was presided by
Lala Lajpat Rai. In December 1920, Congress met once again in the Nagpur Session. This time the
differences of CR Das had melted away. He moved the main resolution of Non-cooperation. A
programme of the surrender of titles, the boycott of schools, courts and councils, the boycott of foreign
goods, the promotion maintenance of Hindu-Muslim unity and strict non-violence was adopted.
• In the Lahore session of 1929, the most landmark resolution was that the Nehru Committee Report had
now lapsed and Dominion status will not be acceptable. A Poorna Swarajya Resolution was passed. In
pursuance with this resolution, the Central and Provincial Legislatures had to be boycotted completely and
all the future elections were also to be boycotted. A Programme of the Civil Disobedience was to be
launched.
• On 8 August 1942 at the All-India Congress Committee session in Bombay, Mohandas Karamchand
Gandhi launched the 'Quit India' movement. The next day, Gandhi, Nehru and many other leaders of the
Indian National Congress were arrested by the British Government. Disorderly and non-violent
demonstrations took place throughout the country in the following days.

Q 47.D
• Post Non Cooperation movement, a question arose in Indian National Congress about the future course of
action, i.e. – whether to get politically involved in governance or not. The sudden withdrawal of Non
Cooperation Movement lead to dissatisfaction all around, it leads to the diffusion of the gained energies in
different ways-
• Indian National Congress was divided into two ideologies, the Swarajist and the No Changers, based on
the council participation or boycott. Swarajist’s main aim was to end the boycott of the council, whereas
No-Changers argued to continue the boycott.
• Swarajists were further divided into Responsivists and Non-Responsivists:
• Responsivists' like Madan Mohan Malviya, Lala Lajpat Rai and N. C. Kelkar offered to cooperate
with the governments to secure the so-called Hindu interests. Hence option (d) is the correct answer.

16 www.visionias.in ©Vision IAS

Google it:- https://upscpdf.com


https://t.me/UPSC_PDF Download From > https://upscpdf.com https://t.me/UPSC_PDF

Q 48.B
• Tulsidas (1497-1623), also known as Goswami Tulsidas, was a Ramanandi Vaishnava saint and poet.
He was a contemporary of Akbar (reigned 1556–1605).
o He was renowned for his devotion to the deity Rama. He wrote several popular works
in Sanskrit and Awadhi, but is best known as the author of the epic Ramcharitmanas, a retelling of
the Sanskrit Ramayana based on Rama's life in the vernacular Awadhi. Hence option (b) is the
correct answer.
• The Ramayana was originally written by Valmiki in Sanskrit and was accessible only to scholars.
Tulsidas brought it to the masses when he wrote Ramcharitmanas in Awadhi, which is a dialect of Hindi.
• Tulsidas started the Ramlila plays, a folk-theatre adaption of the Ramayana
• He has been acclaimed as one of the greatest poets in Hindi, Indian, and world literature. The impact of
Tulsidas and his works on the art, culture and society in India is widespread and is seen to date in
vernacular language, Ramlila plays, Hindustani classical music, popular music, and television series
• Tulsidas Jayanti was recently celebrated on 27th July.
• Ved Vyāsa is the author of the Mahabharata, Vedas and Puranas. The festival of Guru Purnima is
dedicated to him. Exact date of his birth not known but he is believed to have lived around 3rd millennium
BCE.
• Kamban (1180–1250 CE) was a medieval Tamil Hindu poet and the author of the Ramavataram,
popularly known as Kambaramayanam, Tamil version of the epic Ramayana.

Q 49.C
Ryotwari System
• The system was first introduced in Tamil Nadu and later extended to Maharashtra, Berar, East
Punjab, Coorg and Assam. The system was introduced by Sir Thomas Munro in 1820.
• In this system, the responsibility of paying the rent lied with the individual cultivator called
“Ryot”. There existed no intermediaries between the government and the individual
cultivator. Under the Ryotwari system, the land revenue was paid by the farmers directly to the
state. Hence only option (c) is not correct.
• The settlement under the Ryotwari system was not made permanent. It was revised periodically after
20-30 years when the revenue demand was usually raised, which was not like in the case of
Permanent Settlement in Bengal.
• In this system, the peasants or cultivators were regarded as the owners of the land. They had ownership
rights, could sell, mortgage or gift the land. The taxes were directly collected by the government from the
peasants.

Q 50.C
• The Code on Wages, 2019 seeks to regulate wage and bonus payments in all employments where any
industry, trade, business, or manufacture is carried out. The Code replaces the following four laws: (i) the
Payment of Wages Act, 1936, (ii) the Minimum Wages Act, 1948, (iii) the Payment of Bonus Act, 1965,
and (iv) the Equal Remuneration Act, 1976.
• Coverage: The Code will apply to all employees. The central government will make wage-related
decisions for employments such as railways, mines, and oil fields, among others. State governments will
make decisions for all other employments.
• Floor wage: According to the Code, the central government will fix a floor wage, taking into account the
living standards of workers. Further, it may set different floor wages for different geographical
areas. Hence, statement 1 is correct.
• Before fixing the floor wage, the central government may obtain the advice of the Central Advisory Board
and may consult with state governments.
• The minimum wages decided by the central or state governments must be higher than the floor wage. In
case the existing minimum wages fixed by the central or state governments are higher than the floor wage,
they cannot reduce the minimum wages.
• Overtime: The central or state government may fix the number of hours that constitute a normal working
day. In case employees work in excess of a normal working day, they will be entitled to overtime
wage, which must be at least twice the normal rate of wages. Hence, statement 2 is correct.
• Payment of wages: Wages will be paid in (i) coins, (ii) currency notes, (iii) by cheque, (iv) by crediting to
the bank account, or (v) through electronic mode. The wage period will be fixed by the employer as
either: (i) daily, (ii) weekly, (iii) fortnightly, or (iv) monthly. Hence, statement 3 is not correct.
• The Union Labour and Employment Ministry recently published the draft rules framed for the
implementation of the Code on Wages Act, 2019. According to the draft rules, the basis for calculating the
17 www.visionias.in ©Vision IAS

Google it:- https://upscpdf.com


https://t.me/UPSC_PDF Download From > https://upscpdf.com https://t.me/UPSC_PDF

minimum wage would be a standard working-class family of one earning worker, a spouse and two
children, a net intake of 2,700 calories per day each, 66 metres of cloth per year, rent expenditure equal to
10% of the food and clothing expenditure, fuel, electricity and other miscellaneous expenses of 20% of
minimum wage and expenditure on children’s education, medical care, recreation and contingencies
amounting to 25% of minimum wage.

Q 51.C
• The Pitt’s India Act, 1784 also called the East India Company Act, 1784 was passed by the British
Parliament to correct the defects of the Regulating Act of 1773. This act resulted in dual control of
British possessions in India by the British government and the Company with the final authority
resting with the government.
• For political matters, the Board of Control was created and for commercial affairs, the Court of Directors
was appointed. Hence statement 3 is correct.
o The Board of Control took care of civil and military affairs. It comprised of 6 people:
o Secretary of State (Board President)
o Chancellor of the Exchequer
o Four Privy Councillors
• In this dual system of control, the company was represented by the Court of Directors and the British
government by the Board of Control.
• For the first time, the term ‘British possessions in India’ was used. Hence statement 1 is correct.
• This act gave the British government supreme control over Indian administration.
• The company monopoly of Indian and Chinese trade remained intact. Hence statement 2 is correct.
• The right to appoint and transfer British officials was retained by the company.

Q 52.B
• The Ministry of Social Justice and Empowerment recently issued a notification publishing the draft
Transgender Persons (Protection of Rights) Rules, 2020. The Transgender Persons (Protection of
Rights) Act, 2019, was passed by Parliament on November 26 and given Presidential assent on
December 5, 2019.
• The Act aims to end discrimination against transgender persons in accessing education, employment and
healthcare.
• It allows self-perception of gender identity. But it mandates that each person would have to be
recognised as ‘transgender’ on the basis of a certificate of identity issued by a district
magistrate. Hence, statement 1 is not correct.
• It does not offer reservations for employment and education. Hence, statement 2 is not correct.
• The Act provides that the Central Government shall by notification constitute a National Council for
Transgender Persons. The NCT will consist of: (i) Union Minister for Social Justice (Chairperson);
(ii) Minister of State for Social Justice (Vice- Chairperson); (iii) Secretary of the Ministry of Social
Justice; (iv) one representative from ministries including Health, Home Affairs, and Human Resources
Development. Other members include representatives of the NITI Aayog, and the National Human Rights
Commission. State governments will also be represented. The Council will also consist of five members
from the transgender community and five experts from non-governmental organisations. Hence,
statement 3 is correct.

Q 53.B
• The Turtle Survival Alliance (TSA) was formed in 2001 as an International Union for Conservation of
Nature (IUCN) partnership for sustainable captive management of freshwater turtles and tortoises. Hence,
statement 1 is not correct.
• It arose in response to the rampant and unsustainable harvest of Asian turtle populations to supply Chinese
markets, a situation known as the Asian Turtle Crisis.TSA’s conservation actions utilize a three-pronged
approach:
o Restoring populations in the wild where possible
o Securing species in captivity through assurance colonies; and
o Building the capacity to restore, secure and conserve species within their range country.
• It recently developed a mobile application, KURMA, in association with the Indian Turtle Conservation
Action Network (ITCAN) and Wildlife Conservation Society-India. Hence, statement 2 is correct.
• It will not only provides users a database to identify a species but also provides the location of the nearest
rescue center for turtles across the country.

18 www.visionias.in ©Vision IAS

Google it:- https://upscpdf.com


https://t.me/UPSC_PDF Download From > https://upscpdf.com https://t.me/UPSC_PDF

• It serves as a digital database, with a built-in digital field guide covering 29 species of freshwater turtles
and tortoises of India, and information on turtle identification, distribution, vernacular names, and threats.
Tortoise and freshwater turtles are among the most trafficked in the country.

Q 54.C
• A powerful left-wing group developed in India in the late 1920s and 1930s contributing to the
radicalization of the national movement. Jawaharlal Nehru played crucial role in imparting a socialist
vision to the national movement and who became the symbol of socialism and socialist ideas in India after
1929.
• In 1928, Jawaharlal Nehru joined hands with Subhas Chandra Bose and Srinivasa Iyengar to
organize the Independence for India League to fight for complete independence, ‘a socialist revision of
the economic structure of society' and became its general secretary. Srinivas Iyengar was the first
President.
• Hence option (c) is the correct answer.

Q 55.C
• An English association or company to trade with the East was formed in 1599 under the auspices of a
group of merchants known as the Merchant Adventurers. The company was granted a Royal Charter and
the exclusive privilege to trade in the East by Queen Elizabeth on 31 December 1600 and was popularly
known as the East India Company (EIC). From the beginning, it was linked with the monarchy: Queen
Elizabeth was one of the shareholders of the company.
• In 1608 EIC decided to open a factory at Surat on the West coast of India and sent Captain Hawkins to
Jahangir‟s Court to obtain Royal favours. In 1613, Jahangir issued a farman permitting English to
establish a factory permanently at Surat. Therefore, EIC first factory was set up in Surat. Hence,
option (c) is not correct.
• From 1600 to 1757 the East India Company‟s role in India was that of a trading corporation which
brought goods or precious metals into India and exchanged them for Indian goods like textiles, spices,
etc., which it sold abroad. Its profits came primarily from the sale of'Indian goods abroad. Naturally, it
tried constantly to open new markets for Indian goods in Britain and other countries. Thereby, it
increased the export of Indian manufactures and thus encouraged their production.

Q 56.C
• Cornwallis, the second Governor-general of Bengal introduced the system of Permanent Settlement in
1793. It was introduced in provinces of Bengal, Bihar, Orissa and Varanasi. Hence statement 1 is not
correct.
• Under this system, zamindars who earlier only had the right to collect revenue, were not only to act as the
agents of the government in collecting land revenue from the peasants, but also to become owners of the
entire land under their zamindaris. Hence statement 2 is correct.
• The realized amount would be divided into 11 parts. 1/11 of the share belongs to Zamindars and 10/11 of
the share belongs to East India Company.
• Zamindar's right of ownership was made hereditary and transferable. Hence statement 3 is correct.

Q 57.D
• The British imposed a policy of one-way free trade on India after 1813, i.e. the duties on the British
exports into India was reduced to a great extent, so much so that the British goods became cheaper in
the local Indian markets and also, the Indian made handicrafts which initially had high demand in the
western world was reduced by imposing higher import duties there, which made Indian goods look
more costly. Hence statement 1 is correct.
• The British goods were produced on a mass scale by powerful steam-operated machines. The Indian
goods could not compete with the import of cheaper and large scale machine produced goods and thus led
to sudden collapse of Indian handicrafts. Hence statement 2 is correct.
• The ruin of Indian industries, particularly rural artisan industries, proceeded even more rapidly once the
railways were built. The railways enabled British manufacturers to reach and uproot the traditional
industries in the remotest villages of the country. So all the then British Economic policies favoured the
British merchants and there was a clear lack of support by the British government towards its
colonial people. Hence statement 3 is correct.

19 www.visionias.in ©Vision IAS

Google it:- https://upscpdf.com


https://t.me/UPSC_PDF Download From > https://upscpdf.com https://t.me/UPSC_PDF

Q 58.A
• Lichens are symbiotic associations of a fungus and algae or cyanobacterium and occur as crusty patches
or bushy growths on trees, rocks, and bare ground. The names given to lichens strictly refer to the fungal
partner; the algae have separate names. Hence statement 1 is not correct.
• Like all fungi, lichen fungi require carbon as a food source; this is provided by their symbiotic algae
and/or cyanobacteria, that are photosynthetic
• Lichens are widely used as environmental indicators or bio-indicators. If air is very badly polluted with
sulfur dioxide there may be no lichens present, just green algae may be found. If the air is clean, shrubby,
hairy, and leafy lichens become abundant. Hence statement 2 is correct.
• Lichens are very sensitive to sulfur dioxide because their efficient absorption systems result in the rapid
accumulation of sulfur when exposed to high levels of sulfur dioxide pollution. The algal partner seems to
be most affected by the sulfur dioxide; chlorophyll is destroyed and photosynthesis is inhibited.
• Uttarakhand state forest department has recently developed the country's first lichen park in Munsiyari
area of Pithoragarh district. The park, spread over 1.5 acres, is home to over 80 species of lichens
collected from across the Himalayan state. Hence statement 3 is not correct.

Q 59.D
• To make up for the paucity of expenditure on education, the officials had recourse to the so-called
"downward filtration theory". Since the allocated funds could educate only a handful of Indians, it
was decided to spend them in educating a few persons from the upper and middle classes who were
expected to assume the task of educating the masses and spreading modern ideas among them.
Education and modem ideas were thus supposed to filter or radiate downwards from the upper
classes.

Q 60.C
• Kaziranga National Park
• The park was established in 1905 as a reserve forest area. In 1950, it was renamed as Kaziranga Wildlife
Sanctuary. In 1974, the GoI declared the area as National Park.
• In 1985, UNESCO declared the park as UNESCO World Heritage Site. In 2006, the Government of
India declared the park as Tiger Reserve.
• Kaziranga is the biggest habitat of one-horned rhinos. The park has around 2,400 rhinos (more than half
of the total population) and 121 tigers.
• The one-horned rhinoceros is the largest of the three Asian rhinos. The other two are Sumatran Rhinos
and Javan Rhinos
• In the recent floods in the state, around 9 Rhinos were killed. The floods also killed 82 hog deers, two
swamp deers and four wild buffalos and seven wild boars.
• India has collaborated with four countries to protect Asian Rhinos. They are Nepal, Bhutan, Malaysia and
Indonesia. The countries signed the “New Delhi Declaration on Asian Rhinos 2019” to conserve the
rhinos.

Q 61.D
• Sahadaran Ayyapan: was a social reformer, thinker, rationalist, journalist, and politician from Kerala. A
vocal follower of Sree Narayana Guru, he was associated with a number of events related to the Kerala
reformation movement and was the organizer of Misra Bojana in Cherai in 1917. He founded Sahodara
Sangam, and the journal Sahodaran and was the founder editor of the magazine Yukthivadhi. He
changed the call of his guru to - "no religion, no caste and no God for mankind." Hence option (d) is
the correct answer.
• Sri Narayana Guru: was a spiritual leader and social reformer in India. He led a reform movement
against the injustice in the caste-ridden society of Kerala in order to promote spiritual enlightenment and
social equality. It was he who propagated the motto, "One Caste, One Religion, One God for All."
• Raja Rammohan Roy: He is rightly regarded as the first great leader of modern India. He represented a
synthesis of thought of East and West and wanted the people of India to accept the rational and scientific
approach and the principle of human dignity and social equality of all men and women. He also wanted
the introduction of modern capitalism and industry in the country.
• Swami Dayanand: was an Indian philosopher, social leader and founder of the Arya Samaj, a reform
movement of the Vedic dharma. He was the first to give the call for Swaraj as "India for Indians" in 1876,
a call later taken up by Lokmanya Tilak.

20 www.visionias.in ©Vision IAS

Google it:- https://upscpdf.com


https://t.me/UPSC_PDF Download From > https://upscpdf.com https://t.me/UPSC_PDF

Q 62.B
• Statement 2 is correct: The revolt began in Meerut, 36 miles from Delhi on 10 May 1857, and then
gathering force rapidly it cut across Northern India like a sword. It began as a mutiny (resulting from the
discontent of the soldiers), but soon it engulfed wide regions and the civilians. It, thus, turned out to be
much more than just sepoy discontent. It was the outcome of the years of accumulated grievances of the
civilians agains the Company rule.
• Statement 3 is correct: Bahadur Shah is most remembered as the last Mughal Emperor in India, and for
his role in the Indian Revolt of 1857. It would be correct to say that he did not have any active role in it,
rather he was almost like a pawn in the hands of the sepoys, who proclaimed Bahadur Shah as the
Emperor of Hindustan. They wanted a figurehead to rally around and he was their choice.
• Statement 1 is not correct: Most of the rulers of the princely states did not take part in the revolt. These
included the Sindhia of Gwalior, the Holkar of Indore, the rulers of Patiala, Sindh, and other Sikh
chieftains and the Maharaja of Kashmir.

Q 63.A
• Ripon's Resolution of 1882: The Government of Ripon desired the provincial governments to apply in
case of local bodies the same principle of financial decentralisation which Lord Mayo's Government had
begun towards them. Hence option (a) is the correct answer.
The main points of the resolution were as follows:
• Development of local bodies advocated to improve the administration and as an instrument of political
and popular education; Policy of administrating local affairs through urban and rural local bodies charged
with definite duties and entrusted with suitable sources of revenues
• Non-officials to be in majority in these bodies, who could be elected if the officials thought that it was
possible to introduce elections
• Non-officials to act as chairpersons to these bodies
• Official interference to be reduced to the minimum and to be exercised to revise and check the acts of
local bodies, but not to dictate policies
• Official executive sanction required in certain cases, such as raising of loans, alienation of municipal
property, imposition of new taxes, undertaking works costing more than a prescribed sum, framing rules
and bye-laws, etc. In pursuance of this resolution many Acts were passed between 1883 and 1885 which
greatly altered the constitution, powers and functions of municipal bodies in India. But, an era of effective
local self-governing bodies was still a dream unfulfilled.

Q 64.C
• 5 March 1931, the Gandhi-Irwin Pact was signed by Gandhiji on behalf of the Congress and by Lord
Irwin on behalf of the Government.
• The pact placed the Congress on an equal footing with the Government. The terms of the agreement
included
o the immediate release of all political prisoners not convicted for violence,
o the remission of all fines not yet collected,
o the return of confiscated lands not yet sold to third parties, and
o lenient treatment for those government employees who had resigned.
• The Government also conceded the right to make salt for consumption to villages along the coast, as
also the right to peaceful and non-aggressive picketing.
• Congress demand for a public inquiry into police excesses was not accepted, but Gandhiji’s insistent
request for an inquiry was recorded in the agreement. The Congress, on its part, agreed to discontinue the
Civil Disobedience Movement. It was also understood that Congress would participate in the next
Round Table Conference.
• Hence option (c) is the correct answer.

Q 65.B
• The Vernacular Press Act was designed to 'better control' the vernacular press (and not English
newspapers) and effectively punish and repress seditious writing.
• There was strong public opinion against the imperialistic policies of Lytton, compounded by terrible
famine(1876-77), on the one hand, and lavish expenditure on the imperial Delhi Durbar, on the other. The
vernacular press was critical of the Government.
o The district magistrate was empowered to call upon the printer and publisher of any vernacular
newspaper to enter into a bond with the Government undertaking not to cause disaffection against the
Government or antipathy between persons of different groups.

21 www.visionias.in ©Vision IAS

Google it:- https://upscpdf.com


https://t.me/UPSC_PDF Download From > https://upscpdf.com https://t.me/UPSC_PDF

Google it:- https://upscpdf.com


https://t.me/UPSC_PDF Download From > https://upscpdf.com https://t.me/UPSC_PDF

Google it:- https://upscpdf.com


https://t.me/UPSC_PDF Download From > https://upscpdf.com https://t.me/UPSC_PDF

o Teachers adopt Activity Based Learning and move away from rote learning to competency based
learning.
o Teachers and School heads become aware of new initiatives in school education.
o Transformation of the Heads of Schools into providing academic and administrative leadership for the
schools for fostering new initiatives.

Q 72.B
• The Government and the World Bank recently signed a 750 million US dollar agreement for the MSME
Emergency Response Programme. Hence statement 1 is not correct.
• It aims at supporting increased flow of finance into the hands of micro, small, and medium enterprises
(MSMEs) which are severely impacted by the COVID-19 crisis. Hence statement 2 is correct.
• The World Bank’s MSME Emergency Response Programme will address the immediate liquidity and
credit needs of some 1.5 million viable MSMEs to help them withstand the impact of the current shock
and protect millions of jobs.

Q 73.D
• Stars as per the known mechanisms of evolution destroy lithium as they evolve into red giants. Planets
were known to have more lithium than their stars-as is the case with the Earth-Sun Pair.
• However, some stars were found that were lithium-rich leading to a contradiction in the earlier
understanding. This contradiction has been solved by the recent research by the Indian scientist.
• According to research, when stars grew beyond their red giant stage into the red clump stage, they
produce lithium in what is called as helium flash and this is what enriches them with lithium.
• Helium flash occurs at the late stage of a star's evolution as helium accumulates at its core and
causes its temperature and pressure to rise. This state as called the red clump stage.
• Fusion reactor, also called fusion power plant or thermonuclear reactor, a device to produce electrical
power from the energy released in a nuclear fusion reaction. The use of nuclear fusion reactions for
electricity generation remains theoretical.
• A comet tail—and coma—are features visible in comets when they are illuminated by the Sun and may
become visible from Earth when a comet passes through the inner Solar System.
• Aurora, sometimes referred to as polar lights, northern lights, or southern lights, is a natural light display
in the Earth's sky, predominantly seen in the high-latitude regions. Auroras are the result of disturbances
in the magnetosphere caused by solar wind.

Q 74.A
• Jatindra Nath Das, also known as Jatin Das, was a revolutionary and Independence activist. He joined
the Anushilan Samiti, a revolutionary group in Bengal, and also participated in Mahatma Gandhi's
Non-Cooperation movement in 1921, at the tender age of just 17.
• While pursuing BA at Vidyasagar College in Calcutta, in November 1925, he was arrested for political
activities and was jailed at the Mymensingh Central Jail, where he resorted to hunger strike to protest
against the ill-treatment meted out to political prisoners.
• On 8 April, 1929, Bhagat Singh and Batukeswar Dutt of the Hindustan Socialist Republican Army
(HSRA) threw harmless bombs in the Central Legislative Assembly and were arrested.
• On June 14, 1929, Jatin Das was arrested for revolutionary activities and was imprisoned in Lahore jail
along with Bhagat Singh and others to be tried under the supplementary Lahore Conspiracy Case.
• In jail, the members of the HSRA went on a prolonged hunger strike demanding better treatment for
political prisoners, and in September the death of one of them, Jatin Das, on the 64th day of the hunger
strike led to some of the biggest demonstrations the country had ever witnessed. Hence option (a) is the
correct answer.

Q 75.B
• During the 1870s and 1880s, large parts of Eastern Bengal witnessed agrarian unrest caused by
oppressive practices of the zamindars. The zamindars resorted to enhanced rents beyond legal limits and
prevented the tenants from acquiring occupancy rights under Act X of 1859.
• To achieve their ends, the zamindars resorted to forcible evictions, seizure of cattle and crops and
prolonged, costly litigation in courts where the poor peasant found himself at a disadvantage. Having had
enough of the oppressive regime, the peasants of Yusufshahi Pargana in Pabna district (now in
Bangladesh) formed an agrarian league to resist the demands of the zamindars. The league organised
a rent strike—the ryots refused to pay the enhanced rents, challenging the zamindars in the
courts. Funds were raised by ryots to fight the court cases. The struggles spread throughout Pabna and to

24 www.visionias.in ©Vision IAS

Google it:- https://upscpdf.com


https://t.me/UPSC_PDF Download From > https://upscpdf.com https://t.me/UPSC_PDF

other districts of East Bengal. The main form of struggle was that of legal resistance; there was very
little violence. Hence option (b) is the correct answer.
• Though the peasant discontent continued to linger on till 1885, most of the cases had been solved,
partially through official persuasion and partially because of zamindars’ fears. Many peasants were able to
acquire occupancy rights and resist enhanced rents. The government also promised to undertake
legislation to protect the tenants from the worst aspects of zamindari oppression. to fulfill the promise, the
Government passed the Bengal Tenancy Act.

Q 76.B
• Immense intellectual and cultural stirrings characterized nineteenth-century India. Both, the impact of
modern Western culture and consciousness of defeat by a foreign power gave birth to a new
awakening. The socio intellectual revolution that took place in the fields of social reforms is often known
as the Indian Renaissance. Such social reform movement in the nineteenth century was initiated by people
like Raja Ram Mohun Roy (the founder of Brahmo Samaj), Ishwar Chand Vidya Sagar, etc
• Tattwabodhini Sabha: On 6 October 1839 Debendranath Tagore established Tattvaranjini Sabha which
was shortly thereafter renamed the Tattwabodhini (Truth-seekers) Sabha, as a splinter group of the
Brahmo Samaj. Among its first members were the "two giants of Hindu reformation and Bengal
Renaissance, Akshay Kumar Datta and Ishwar Chandra Vidyasagar. In 1859, the Tatvabodhini Sabha was
dissolved back into the Brahmo Samaj by Debendranath Tagore.
• The main objective of the Sabha was to promote a more rational and humanist form of Hinduism based on
the Vedanta, the Upanishads that form the last part of the Vedas. The Tattwabodhini Sabha aimed to
shield themselves and their reformed faith from criticism by distancing themselves from this
'outdated' version.
• Rehnumai Mazdayasan Sabha: In 1851, the Rehnumai Mazdayasan Sabha or Religious Reform
Association was started by Naoroji Furdonji, Dadabhai Naoroji, S.S.Bengalee, and others. It campaigned
against orthodoxy and initiated the modernization of Parsi social customs regarding the education
of women, marriage, and the social position of women in general. In the course of time, the Parsis
became the most westernized section of Indian society.
• Deoband Movement: is a revivalist movement formed in the town of Deoband. The seminary was
founded by Muhammad Qasim Nanautavi, Rashid Ahmad Gangohi, and several other figures in 1866. The
Deobandi movement developed as a reaction to British colonialism which was seen by a group of Indian
scholars to be corrupting Islam and aimed at organizing for Islamic revival and anti-imperialist ideology.
• Hence option (b) is the correct answer.

Q 77.B
• Pandit Ishwar Chandra Vidyasagar was a great scholar and social reformer, who dedicated his entire
life to the cause of social reform. In 1851, he became the Principal of the Sanskrit College,
Calcutta. He opened the gates of this college to non-Brahmin students.
• Though he was a great Sanskrit scholar, his mind was open to the best in Western thoughts, and he came
to represent a happy blend of Indian and Western culture. He waged a long struggle in favor of widow
remarriage. He raised his voice, backed by the weight of immense traditional learning in favour of
widow remarriage in 1855.
• He was also deeply interested in the education of women and as a Government Inspector of Schools, he
organized 35 girl's schools, many of which he ran at his own expense. As a Secretary to Bethune School,
he was one of the pioneers for higher education for women. Hence option (b) is the correct answer.
• Debendranath Tagore (15 May 1817 – 19 January 1905) was a Hindu philosopher and religious
reformer, active in the Brahmo Samaj (“Society of Brahma,” also translated as “Society of God”). He was
the founder in 1848 of the Brahmo religion, which today is synonymous with Brahmoism.
• Jagannath Shankarsheth Murkute (10 February 1803 – 31 July 1865) was an Indian philanthropist and
educationalist. Unlike his forefathers, who performed priestly duties for his community, he engaged in
commerce and soon developed a reputation as a very reliable businessman. He became one of the
founders of the School Society and the Native School of Bombay.
• Karsandas Mulji (25 July 1832 – 28 August 1871) was a Gujarati language journalist, writer and social
reformer from IndiaBorn to a family belonging to the Kapol Caste, a trading caste of western India, he
was repudiated by his family because of his views on widow remarriage. He became a vernacular
schoolmaster and started Satyaprakash, a weekly in Gujarati, in which he attacked what he perceived to be
the immoralities of the Maharajas or hereditary high priests of the Pushtimarg Vaishnavism,

25 www.visionias.in ©Vision IAS

Google it:- https://upscpdf.com


https://t.me/UPSC_PDF Download From > https://upscpdf.com https://t.me/UPSC_PDF

Q 78.B
• Moderates believed in the policy of settlement of minor issues with the government by deliberations. But
the extremists believed in agitation, strikes, and boycotts. Nationalists led by Lokmanya Tilak agitated
against the Moderates.
• The conflict between these two sections became visible at the end of Congress' Banaras Session (1905).
The difference between moderates and extremists widened in Congress' Calcutta Session of (1906) and
attempts were made to select one of them as the president. The moderates opposed the resolutions on
Swaraj, Swadeshi, Boycott of foreign goods, and National Education and requested to withdraw from the
policy laid down in the Calcutta session. But the extremists were not ready to do so.
• In 1907, Surat Session, the two main objectives placed by the extremists were:
o Demand for the resolution of Swaraj
o Lala Lajpat Rai to be made the President of the INC
• These two demands were not acceptable to the moderates. Thus, instead of Lala Lajpat Rai, the moderates
supported the idea of Rash Behari Ghosh as the President. The moderate leaders have captured the
machinery of the congress excluded the militant extremist elements from it. The congress split into two
separate groups of moderates and extremists. Hence option (b) is the correct answer.

Q 79.A
• Harijan Sevak Sangh was founded by Mahatma Gandhi in 1932 when he left Sabarmati and decided to
settle in Wardha till Swaraj and eradication of untouchability was achieved. He undertook intensive
"Harijan Tour" traveling across the country and propagating the removal of untouchability in all its forms
and practices. He collected money under this organization.
• After the Second Round Table Conference, the British government agreed to give Communal Award to
the depressed classes on the request of B. R. Ambedkar. Gandhi opposed the government's decision which
he considered would divide the Hindu society and subsequently went on to the indefinite fast in Yerwada
Jail. He ended his fast after signed Poona Pact with Ambedkar on 24 September 1932. On 30 September,
Gandhi founded All India Anti Untouchability League, to remove untouchability in the society,
which later renamed as Harijan Sevak Sangh ("Servants of Untouchables Society").
• At the time industrialist, Ghanshyam Das Birla was its founding president with Amritlal Takkar as
its Secretary.

Q 80.A
• The Government of India Act, 1919 had a provision that a commission would be appointed ten years
from the date to study the progress of the governance scheme and suggest new steps. In 1927, the
Government of Britain appointed an all-white, seven-member Indian Statutory Commission, popularly
known as the Simon Commission after its Chairman.
• That no Indian should be thought fit to serve on a body that claimed the right to decide the political
future of India was an insult to every Indian. Hence statement 1 is correct.
• The call for a boycott of the Commission was endorsed by the Liberal Federation led by Tej Bahadur
Sapru, by the Hindu Mahasabha; and by Mohammed Ali Jinnah carrying the majority of Muslim League
with him. However, it was Indian National Congress, however, that turned the boycott into a popular
movement. During the Anti-Simon Commission protests, Lala Lajpat Rai was attacked by the police
and shortly thereafter he died.
• The Simon Commission published a two-volume report in May 1930. The Commission contained no
mention of Dominion Status and was in other ways also a regressive document. It proposed the abolition
of dyarchy and the establishment of representative government in the provinces which should be given
autonomy. It said that the governor should have discretionary power in relation to internal security and
administrative powers to protect the different communities. Hence statement 2 is not correct.

Q 81.B
• The Indian army was carefully reorganized after 1858. Some changes were made necessary by the transfer
of power to the Crown. These were:
o The domination of the army by its European branch was carefully guaranteed.
o The proportion of Europeans to Indians in the army was raised and fixed at one to two in the Bengal
Army and two to five in the Madras and Bombay armies. Hence statement 1 is not correct.
o The European troops were kept in key geographical and military positions. The crucial branches of
the army like artillery and, later in the 20th century, tanks, and armored corps were put exclusively in
European hands. Hence statement 2 is correct.

26 www.visionias.in ©Vision IAS

Google it:- https://upscpdf.com


https://t.me/UPSC_PDF Download From > https://upscpdf.com https://t.me/UPSC_PDF

o The older policy of excluding Indians from the officer corps was strictly maintained. Till 1914, no
Indian could rise higher than the rank of a subedar.
o The organization of the Indian section of the army was based on the policy of “balance and
counterpoise” or “divide and rule” so as to prevent its chances of uniting again in an anti-British
uprising.
o Discrimination on the basis of caste, region, and religion was practiced, in recruitment to the army.
o A fiction was created that Indians consisted of “martial” and “non-martial” classes.
o Soldiers from Avadh, Bihar, Central India, and South India who had first helped the British conquer
India but had later taken part in the Revolt of 1857, were declared to be non-martial. They were no
longer taken in the army on a large scale.
o The Sikhs, Gurkhas, and Pathans, who had assisted in the suppression of the Revolt, were declared to
be martial and were recruited in large numbers.
o The Indian regiments were made a mixture of various castes’ and groups’ which were so placed as to
balance each other.
o Communal, caste, tribal, and regional loyalties were encouraged among the soldiers, so that the
sentiment of nationalism would not grow among them.
Q 82.B
• The left-wing of the Congress was not content with the Nehru report of 1928 as it did not mention
complete Independence but wanted only a dominion status.
• The Congress has resolved that if the British Parliament accepts Nehru report by 31 December 1929,
Congress would support the report as it is. Otherwise, Congress would insist on 'Complete Independence'.
No positive reply came from the British. This was followed by the Lahore Session of Congress.
• Jawaharlal Nehru was nominated the President for the Lahore session of the Congress (December
1929) mainly due to Gandhi’s backing. The following major decisions were taken at the Lahore session.
o The Round Table Conference was to be boycotted.
o Complete independence (Poorna Swaraj) was declared as the aim of Congress.
o Congress Working Committee was authorised to launch a programme of civil disobedience including
non-payment of taxes and all members of legislatures were asked to resign their seats.
o January 26, 1930, was fixed as the first Independence (Swarajya) Day, to be celebrated
everywhere.
• The resolutions on Fundamental Rights and on the National Economic Programme were adopted in
the Karachi Session of the Congress in 1931.
• The Two nation theory was propounded in the Lahore Session of the Muslim League in 1940.
• Hence option (b) is the correct answer.

Q 83.B
• In Bengal during mid 19th century, the indigo planters, nearly all Europeans, exploited the local
peasants by forcing them to grow indigo on their lands instead of the more paying crops like rice. There
were two mains systems of Indigo cultivation - nij and ryoti. In nij cultivation, the planter produced
indigo in lands that planter directly controlled. Under the ryoti system, the planters forced the ryots to sign
a contract, an agreement (satta) to take advance sums and enter into fraudulent contracts which were then
used against the peasants. Hence statement 1 is not correct.
• The anger of the peasants exploded in 1859 when, led by Digambar Biswas and Bishnu Biswas of Nadia
district, they decided not to grow indigo under duress and resisted the physical pressure of the planters and
their lathiyals (retainers). The ryots replied by going on a rent strike by refusing to pay the enhanced
rents and by physically resisting the attempts to evict them. Gradually, they learned to use the legal
machinery and initiated legal action supported by fund collection.
• The intelligentsia of Bengal organized a powerful campaign in support of the rebellious peasantry. Harish
Chandra Mukherji, editor of the Hindoo Patriot published regular reports from his correspondents in
the rural areas on planters’ oppression, officials’ partisanship and peasant resistance. Din Bandhu
Mitra’s play, Neel Darpan, was to gain great fame for vividly portraying the oppression by the planters.
In Anandmath novel of Bankim Chandra Chattopadhyay Sanyasi rebellion was described.
• The indigo peasants imagined that the British government would support them in their struggle
against the planters. The revolt was mainly directed against the planters not the British. Hence
statement 2 is not correct.
• The Government appointed an Indigo Commission to inquire into the problem of indigo
cultivation. The Commission found that the indigo cultivation system was oppressive in nature
especially because of the system of advances. The Commission held the planters guilty and criticised them
for the coercive methods they used with indigo cultivators. Hence statement 3 is correct.
27 www.visionias.in ©Vision IAS

Google it:- https://upscpdf.com


https://t.me/UPSC_PDF Download From > https://upscpdf.com https://t.me/UPSC_PDF

Q 84.D
• The Congress, for the first time at its Nagpur Session in 1920, enunciated its policy towards the
peoples’ movement in the Princely States. It called upon the Princes to grant full responsible
government in their States. However, it was pointed out that though the people belonging to the States
could enroll themselves as members of the Congress, they could not initiate political activity in the
State in the name of the Congress. They could carry on political activity in their individual capacity as
members of the local Praja Mandals. Hence statement 1 is not correct.
• The years 1938-39, in fact, stand out as years of a new awakening in the Indian States and were witness to
a large number of movements demanding responsible government and other reforms. Praja mandals
mushroomed in many States that had earlier no such organizations.
• Whereas, even in the Haripura session in 1938, the Congress had reiterated its policy that
movements in the States should not be launched in the name of the Congressbut should rely on their
own independent strength and fight through local organizations, a few months later, on seeing the
new spirit that was abroad among the people and their capacity to struggle. Gandhiji and the Congress
changed their attitude on this question. The radicals and socialists in the Congress, as well as political
workers in the States, had in any case been pressing for this change for quite some time.
• Following upon this, the Congress at Tripuri in March 1939 passed a resolution enunciating its new
policy: ‘The great awakening that is taking place among the people of the States may lead to relaxation, or
to complete removal of the restraint which the Congress imposed upon itself, thus resulting in an ever-
increasing identification of the Congress with the States’ peoples’. Also in 1939, the All India States
Peoples' Conference (AISPC) elected Jawaharlal Nehru as its President for the Ludhiana session, thus
setting the seal on the fusion of the movements in Princely India and British India.
• The outbreak of the Second World War brought about adistinct change in the political atmosphere.
Congress Ministriesr esigned, the Government armed itself with the Defence of India Rules, and in the
States as well there was less tolerance of political activity. Things came to a head again in 1942 with the
launching of the Quit India Movement. This time the Congress made no distinction between British
India and the Indian States and the call for struggle was extended to the people of the States. The
people of the States thus formally joined the struggle for Indian independence, and in addition to their
demand for responsible government they asked the British to quit India and demanded that the States
become integral parts of the Indian nation. Hence, statement 2 is not correct.

Q 85.B
• Rajasthan Speaker recently served notices to 19 Congress MLAs asking them why they cannot be
disqualified. The notice was served under the Tenth Schedule of the Constitution, popularly known as the
anti-defection law. Congress in its complaint to the Speaker has accused the rebel MLAs of attempting to
jump parties.
• There are two conditions on which a Speaker can serve an anti-defection notice to a legislator. The first is
when the legislator has voluntarily given up the membership of the party, and the second condition is
when a legislator has disobeyed the whip of the party during a vote.
• Even when the disqualification process is pending, the rebel Congress legislators would still have to abide
by the party whip in case of a trust vote. If they vote against the party or abstain, they could subsequently
be disqualified. However, their vote, even when they disobey the whip, will be counted as it is cast, and
not on party lines. But if they resign as legislators and then jump parties, then the strength of the House
and that of the ruling party will be reduced.
• Resigning as an MLA and subsequently joining another party does not attract disqualification.
Hence, statement 2 is correct.
• In both cases, resignation and disqualification, the member can contest the bye-election. However, the
key difference is that when a legislator resigns, she can be immediately appointed a minister in the
next government and will get six months to get re-elected, but when a member is disqualified, she
cannot be a part of the government till she wins the polls. The difference then is going to polls as a
Minister as opposed to a former MLA, a route that is less preferred. Hence, statement 1 is not correct.

Q 86.C
• When Alivardi Khan died in 1756, Sirajuddaulah became the Nawab of Bengal. The Company was
worried about his power and keen on a puppet ruler who would willingly give trade concessions and other
privileges. So it tried, but without success, to help one of Sirajuddaulah’s rivals become the nawab. An
infuriated Sirajuddaulah asked the Company to stop meddling in the political affairs of his dominion, stop
fortification, and pay the revenues. After negotiations failed, the Nawab marched with 30,000 soldiers to
the English factory at Kassimbazar, captured the Company officials, locked the warehouse, disarmed all
28 www.visionias.in ©Vision IAS

Google it:- https://upscpdf.com


https://t.me/UPSC_PDF Download From > https://upscpdf.com https://t.me/UPSC_PDF

Englishmen, and blockaded English ships. Then he marched to Calcutta to establish control over the
Company’s fort there.On hearing the news of the fall of Calcutta, Company officials in Madras sent forces
under the command of Robert Clive, reinforced by naval fleets. Prolonged negotiations with the Nawab
followed. Finally, in 1757, Robert Clive led the Company’s army against Sirajuddaulah at
Plassey. Hence statement 1 is correct.
• One of the main reasons for the defeat of the Nawab was that the forces led by Mir Jafar, one of
Sirajuddaulah’s commanders, never fought the battle. Clive had managed to secure his support by
promising to make him nawab after crushing Sirajuddaulah.
• The Battle of Plassey became famous because it was the first major victory the Company won in
India.After the defeat at Plassey, Sirajuddaulah was assassinated and Mir Jafar made the nawab of
Bengal. The Company was still unwilling to take over the responsibility of administration. Its prime
objective was the expansion of trade. If this could be done without conquest, through the help of local
rulers who were willing to grant privileges, then territories need not be taken over directly. Hence
statement 2 is correct.

Q 87.B
• Lord Birkenhead, the Conservative Secretary of State responsible for the appointment of the Simon
Commission, had constantly harped on the inability of Indians to formulate a concrete scheme of
constitutional reforms. As an answer to Lord Birkenhead’s challenge, an All Parties Conference met in
1928 and appointed a subcommittee under the chairmanship of Motilal Nehru to draft a constitution.
• Important recommendations:
o This report defined Dominion Status as the form of government desired by India. Hence
statement 3 is not correct.
o It also rejected the principle of separate communal electorates on which previous constitutional
reforms had been based. Seats would be reserved for Muslims at the Centre and in provinces in
which they were in a minority, but not in those where they had a numerical majority. Hence
statement 1 is not correct.
o The Report also recommended universal adult suffrage, equal rights for women, freedom to form
unions, and dissociation of the state from religion in any form. Hence statement 2 is correct.
o Responsible government at the Centre and in provinces
o Provincial councils to have a 5-year tenure, headed by a governor acting on the advice of the
provincial executive council.
• Hence option (b) is the correct answer.

Q 88.C
• Cunningham Circular played a very important role in shaping the freedom struggle in Assam. While the
entire country was getting ready for the start of the Civil Disobedience Movement in 1930, students in
Assam played a key role in the freedom struggle.
• To stem the students' participation in the Civil Disobedience Movement, the 'Cunning Circular' was
implemented in 1930 by the British. This ruling forbade students from participating in political
activities. It forced students and their guardians to furnish assurances of good behaviour. Hence
option (c) is the correct answer.
• Students quit schools in protest and many educational institutions like Kamrup Academy of Guwahati and
Sibsagar Vidyapeeth were established.

Q 89.C
• The Indian Statutory Commission, commonly referred to as the Simon Commission, was a group of seven
English, male British Members of Parliament under the chairmanship of Sir John Simon. The commission
arrived in British India in 1928 to study constitutional reform in Britain's largest and the most important
possession.
• One of its members was Clement Attlee, of the Labour Party, who became committed to Indian
independence by 1934 and achieved that goal as Prime Minister in 1947 in the granting of
independence to India and the creation of Pakistan.
• At the time of introducing the Montagu-Chelmsford Reforms in 1919, the British Government declared
that a commission would be sent to India after ten years to examine the effects and operations of the
constitutional reforms and to suggest more reforms for India.
• In November 1927, the British government appointed the Simon Commission to report on India's
constitutional progress for introducing constitutional reforms, as promised.
• The response in India was immediate and unanimous. That no Indian should be thought fit to serve on
a body that claimed the right to decide the political future of India was an insult that no Indian of
29 www.visionias.in ©Vision IAS

Google it:- https://upscpdf.com


https://t.me/UPSC_PDF Download From > https://upscpdf.com https://t.me/UPSC_PDF

even the most moderate political opinion was willing to swallow. The call for a boycott of the
Commission was endorsed by the Liberal Federation led by Tej Bahadur Sapru, by the Indian
Industrial and Commercial Congress, and by the Hindu Mahasabha. The Muslim League even split
on the issue, Mohammed Ali Jinnah carrying the majority with him in favor of the boycott. Hence
options 1, 3, and 4 are correct.
• In 1927, the Indian National Congress passed a resolution boycotting Simon Commission under the
presidentship of M.A.Ansari, in Madras supported by all.
• The action began as soon as Simon and his friends landed at Bombay on 3 February 1928. That day, all
the major cities and towns observed a complete hartal, and people were out on the streets participating in
mass rallies, processions, and black-flag demonstrations. In Madras, a major clash with the police resulted
in the firing and the death of one person. T. Prakasam symbolized the defiant spirit of the occasion by
baring his chest before the armed policemen who tried in vain to stop him from going to the scene of the
killing. Everywhere that Simon went, Calcutta, Lahore, Lucknow, Vijayawada, Poona, he was
greeted by a sea of black-flags carried by thousands of people with "Go Back Simon" written on
them.
• In 1929, Dr. B.R. Ambedkar made a controversial decision to co-operate with the all-British Simon
Commission which was to look into setting up a responsible Indian Government in India. The
Congress decided to boycott the Commission and drafted its own version of a constitution for free India.
The Congress version had no provisions for the depressed classes. Ambedkar became more skeptical of
Congress's commitment to safeguard the rights of the depressed classes. He represented the "Bahishkrit
Hitakarni Sabha" in Bombay Legislative Council and cooperated with Simon
Commission. Hence option 2 is not correct.

Q 90.D
• Portugal had a monopoly of the highly profitable Eastern trade for nearly a century. In India, she
established her trading settlements at Cochin, Goa, Dlu, and Daman. From the beginning, the Portuguese
combined the use of force with trade. Portugal was, however, incapable of maintaining for long its trade
monopoly or its dominions in the East. By the 18th century, the Portuguese in India lost their commercial
influence, though some of them still carried on trade in their individual capacity and many took to piracy
and robbery. The decline of the Portuguese was brought about by several factors:
o The local advantages gained by the Portuguese in India were reduced with the emergence of powerful
dynasties in Egypt, Persia and North India and the rise of the turbulent Marathas as their immediate
neighbors. (The Marathas captured Salsette and Bassein in 1739 from the Portuguese). Also
their merchants enjoyed much less power and prestige than the landed aristocrats in India.
Hence statement 1 is correct.
o The religious policies of the Portuguese, such as the activities of the Jesuits, gave rise to political
fears. Their antagonism for the Muslims apart, the Portuguese policy of conversion to Christianity
made Hindus also resentful. Hence statement 3 is correct.
o Their dishonest trade practices also evoked a strong reaction. The Portuguese earned notoriety as sea
pirates. Their arrogance and violence brought them the animosity of the rulers of small states and the
imperial Mughals as well.
o The earlier monopoly of knowledge of the sea route to India held by the Portuguese could not
remain a secret forever; soon enough the Dutch and the English, who were learning the skills of
ocean navigation, also learned it and outshined the Portuguese. Hence statement 2 is correct.
o As new trading communities from Europe arrived in India, there began a fierce rivalry among them.
In this struggle, the Portuguese had to give way to the more powerful and enterprising competitors.
The Dutch and the English had greater resources and more compulsions to expand overseas, and they
overcame the Portuguese resistance. One by one, the Portuguese possessions fell to its opponents
Q 91.C
• The essence of British imperialism lay in the subordination of the Indian economy to the British economy.
They operated through the more disguised and complex mechanism of free trade and foreign capital
investment.
• Their policies aimed at transformation of India into supplier of food stuffs and raw materials to the
metropolis, a market for the metropolitan manufacturers, and a field for the investment of British
capital. Hence, all the statements are correct.
• The critics of the economic policies of the British like Dadabhai Naoroji, Mahadev Govind Ranade, RC
Dutt, G.V. Joshi, G. Subramaniya Iyer, G.K.Gokhale, Prithwis Chandra Ray, etc. clearly understood this
and delineated the colonial structure in all its three aspects of domination through trade, industry and
finance.
30 www.visionias.in ©Vision IAS

Google it:- https://upscpdf.com


https://t.me/UPSC_PDF Download From > https://upscpdf.com https://t.me/UPSC_PDF

Q 92.D
• The need for the civil service was felt soon after the Company acquired territories after the Battles of
Plassey (1757) and Buxar (1764).
• Warren Hastings, the then Governor-General of Bengal had created the post of District Collector who was
made in-charge of collecting land revenue. This post was soon abolished on grounds of excessive
concentration of powers and corruption.
• Lord Cornwallis is usually known as the Father of civil services in India. He had introduced the
covenanted civil services and the uncovenanted civil services. Hence statement 1 is correct.
• A special feature of the Indian civil services since the days of Cornwallis was the rigid and complete
exclusion of Indians from it .
• The initial attempt to train the Civil Servants locally was done by Lord Wellesley. Within the campus of
the Fort William, he founded Fort Williams College on 10 July 1800. Hence statement 2 is correct.
• Until 1853, the Court of Directors had the exclusive right to appoint persons in the Company's civil
services. These appointments were a source of privilege and patronage which the Company held on to
very tightly.
• The Charter Act of 1853 provided for an open competitive examination for the recruitment of civil
servants and had deprived the Court of Directors of the power of appointments based on patronage. This
was recommended by a committee headed by Lord Macaulay. The first competitive exam was held in
1855. Hence statement 3 is correct.

Q 93.D
• Bhagat Singh was an Indian socialist revolutionary who is revered as a leader of the young and
masses. Bhagat Singh, born in 1907 and a nephew of the famous revolutionary Ajit Singh, was a
giant of an intellectual. A voracious reader, he was one of the most well-read political leaders of the
time. He had devoured books in the Dwarkadas Library at Lahore on socialism, the Soviet Union, and
revolutionary movements, especially those of Russia, Ireland, and Italy.
• Bhagat Singh had already, before his arrest in 1929, abandoned his belief in terrorism and individual
heroic action. He had turned to Marxism and had come to believe that popular broad-based mass
movements alone could lead to a successful revolution; in other words, the revolution could only be
achieved ‘by the masses for the masses.’ That is why Bhagat Singh helped establish the Punjab
Naujawan Bharat Sabha in 1926 (becoming its founding Secretary), as the open wing of the
revolutionaries.
• The Sabha was to carry out open political work among the youth, peasants, and workers. It was to open
branches in the villages. Under its auspices, Bhagat Singh used to deliver political lectures with the help
of magic lantern slides. Bhagat Singh and Sukhdev also organized the Lahore Students Union for open,
legal work among the students.
• Bhagat Singh was a great innovator in two areas of politics. Being fully and consciously secular, he
understood, more clearly than many of his contemporaries, the danger that communalism posed to
the nation and the national movement. He often told his audience that communalism was as big an
enemy as colonialism.
Political Organizations:
• Hindustan Socialist Republican Army: Post Kakori Case, Hindustan Republic Army faced a major
setback and became a bit inactive. In 1928, in the atmosphere of Simon Commission Resistance,
younger men such as Bejoy Kumar Sinha, Shiv Varma, and Jaidev Kapur in U.P.: Bhagat Singh,
Bhagwati Charan Vohra, and Sukhdev in Punjab set out to reorganize the HRA under the overall
leadership of Chandrashekhar Azad. Simultaneously, they were being influenced by socialist ideas.
Finally, nearly all the major young revolutionaries of northern India met at Ferozeshah Kotla Ground at
Delhi on 9 and 10 September 1928, created a new collective leadership, adopted socialism as their
official goal, and changed the name of the party to the Hindustan Socialist Republican Association
(Army). Bhagat Singh was its first secretary.
• Workers' & Peasants Party: The Workers and Peasants Party (WPP) (also known as the Kirti Kisan
Party) was a political party in India, which worked inside the Indian National Congress in 1925-1929. It
became an important front organization for the Communist Party of India and an influential force in the
Bombay labor movement. Bhagat Singh wrote for Kirti, the journal of the Kirti Kisan Party
("Workers and Peasants Party").

31 www.visionias.in ©Vision IAS

Google it:- https://upscpdf.com


https://t.me/UPSC_PDF Download From > https://upscpdf.com https://t.me/UPSC_PDF

Q 94.A
• Statement 1 is correct and statement 2 is not correct: The Union Cabinet recently gave its approval
to a new pan India Central Sector Scheme- Agriculture Infrastructure Fund. The scheme will
provide a medium and long term debt financing facility for investment in viable projects for post-
harvest management Infrastructure and community farming assets through interest subvention and
financial support. Under the scheme, one lakh crore rupees will be provided by banks and financial
institutions as loans to Primary Agricultural Credit Societies, Marketing Cooperative Societies, Farmer
Producers Organizations, Self Help Group, Farmers, Startups and Centre and State agency among others.
• The duration of the Scheme will be for ten years from Fiscal year 2020 to 2029. Under it, loans will be
disbursed in four years starting with sanction of 10 thousand crore rupees in the current year and 30
thousand crore rupees each crore in next three financial years. The Project by way of facilitating formal
credit to farm and farm processing-based activities is expected to create numerous job opportunities in
rural areas.

Q 95.A
• During the Civil Disobedience Movement, Eastern India became the scene of a new kind of no-tax
campaign: refusal to pay the chowkidara tax. Chowkidars, paid out of the tax levied especially on the
villages, were guards who supplemented the small police force in the rural areas in this region. Hence
statement 1 is correct but statement 3 is not correct.
• They were particularly hated because they acted as spies for the Government and often also as retainers
for the local landlords. The movement against this tax and calling for the resignation of Chowkidars,
and of the influential members of Chowkidari panchayats who appointed the Chowkidars, first
started in Bihar in May 1930 itself, as salt agitation had not much scope due to the land-locked
nature of the province. Hence statement 2 is correct.
• In the Monghyr, Saran, and Bhagalpur districts, for example, the tax was refused, Chowkidars induced to
resign, and social boycott used against those who resisted.
• The Government retaliated by confiscation of property worth hundreds and thousands in lieu of a few
rupees of tax, and by beatings and torture. Matters came to a head in Bihpur in Bhagalpur on May 31
when the police, desperate to assert its fast-eroding authority, occupied the Congress ashram which was
the headquarters of nationalist activity in the area.
• The occupation triggered off daily demonstrations outside the ashram, and a visit by Rajendra Prasad and
Abdul Ban from Patna became the occasion for, a huge mass rally, which was broken up by a lathi charge
in which Rajendra Prasad was injured. As elsewhere, repression further increased the nationalists’
strength, and the police just could not enter the rural areas.
• In Bengal, the onset of the monsoon, which made it difficult to make salt, brought about a shift to anti-
chowkidara and anti-Union Board agitation. Here too, villagers withstood severe repression, losing
thousands of rupees worth of property through confiscation and destruction, and having to hide for days in
forests to escape the wrath of the police.

Q 96.A
• The UNCAT aims to prevent torture and other acts of cruel, inhuman, or degrading treatment or
punishment around the world. The Convention requires states to take effective measures to prevent torture
in any territory under their jurisdiction, and forbids states to transport people to any country where
there is reason to believe they will be tortured. Hence, statement 1 is correct.
• Although India signed the UNCAT in 1997, it is yet to ratify it. In 2010, a Prevention of Torture Bill was
passed by the Lok Sabha, and the Rajya Sabha later sent it to a Select Committee for review in alignment
with the UNCAT. But the Committee’s recommended law, submitted in 2012, never fructified. Hence,
statement 2 is not correct.
• The tragic deaths of P. Jayaraj and J. Benicks, a father-son duo in a small town in Thoothukudi (Tamil
Nadu), under police custody reignited the demand for ratification of UNCAT by India.

Q 97.C
• Khudai Khidmatgar (servants of God) was a predominantly Pashtun nonviolent resistance movement
known for its activism against the British Raj in colonial India. It was based in British India's North-West
Frontier Province (now in Khyber Pakhtunkhwa, Pakistan). Hence option (c) is the correct answer.
• Also called "Red Shirts", this was originally a social reform organisation focusing on education and the
elimination of blood feuds. The movement was led by Abdul Ghaffar Khan, known locally as Bacha
Khan.

32 www.visionias.in ©Vision IAS

Google it:- https://upscpdf.com


• It gradually became more political as its members were being targeted by the British Raj. By 1929 its
leadership was exiled from the province and large numbers were arrested.
• It played an active role in the Civil Disobedience Movement. The atmosphere created by its political
work contributed to the mass upsurge in Peshawar during the movement. The Peshawar demonstrations
are significant because it was here that the soldiers of the Garhwali regiments refused to fire on the
unarmed crowd.

Q 98.A
• API (Active Pharmaceutical Ingredient) means the biologically active ingredients which is contained in
the medicine. For example, an active ingredient to relieve pain is included in a painkiller. This is called
API. A small amount of the active ingredient has an effect, so only a tiny part of the active ingredient is
contained in the medicine. Hence, statement 1 is correct.
• China is the world's largest producer of pharma ingredients and the world’s second-largest pharma
market. India ranks the 3rd largest manufacturer of Active Pharmaceutical Ingredient (API) in the Asia-
Pacific region. Hence, statement 2 is not correct.
• India is currently dependent on China for 70% of its API requirements.
• Under Atmanirbhar Bharat, India is aiming to reduce import dependence and achieve self-reliance. One
such step is the recently proposed hike in import duty from the current 10%.
• Scheme for promotion of bulk drug and production linked incentive scheme have already been launched
to bs the bulk drug in India.

Q 99.B
• Before the Indian nationalist intelligentsia began to associate itself with working-class agitations towards
the end of the 19th century, there were some early attempts at the organized effort to improve the
condition of the workers.
• In 1878, Sorabjee Shapoorji Bengalee tried unsuccessfully to introduce a Bill in the Bombay Legislative
Council to limit the working hours for labour. In Bengal, Sasipada Banerjee, a Brahmo Social
reformer, set up a Workingmen’s Club in 1870 and brought out a monthly journal called Bharat
Sramjeebi (Indian Labour), with the primary idea of educating the workers. In Bombay, Narayan
Meghajee Lokhanday brought out an Anglo-Marathi weekly called Dina-Bandhu (Friend of the Poor) in
1880 and started the Bombay Mill and Millhands’Association in 1890. Hence statements 2 and 3 are
correct.
• However, these philanthropists did not belong to the mainstream of the contemporary national movement.
The early nationalists, in the beginning, paid relatively little attention to the question of
workers despite the truly wretched conditions under which they existed at that time. It is because the
nationalists did not wish to weaken the common struggle against British rule by creating any divisions
within the ranks of the Indian people. Dadabhai Naoroji in 1986 said that Congress must confine itself to
questions in which the entire nation has direct participation, and it must leave the adjustment of social
reforms and other class questions to class Congresses.’ Hence statement 1 is not correct.
• The Home Rule Leagues, the Rowlatt Satyagraha and Non-Cooperation movement led to the resurgence
of working-class activity in the years from 1919 to 1922. The most important development was the
formation of the All India Trade Union Congress (AITUC) in 1920. Lokamanya Tilak was one of the
moving spirits in the formation of the AITUC, which had Lala Lajpat Rai as its first president and
Dewan Chaman Lal as its general secretary.

Q 100.A
• The "Bombay Plan" (1944) is the name commonly given to a World War II-era set of proposals for the
development of the post Independence economy of India. Titled "A Brief Memorandum Outlining a Plan
of Economic Development for India", the signatories of the Plan were Jehangir Ratanji Dadabhoy Tata,
Ghanshyam Das Birla, Ardeshir Dalal, Sri Ram, Kasturbhai Lalbhai, Ardeshir Darabshaw Shroff, Sir
Purshottamdas Thakurdas and John Mathai.
• The Bombay plan argued for comprehensive land reform, including cooperativization of production,
finance and marketing. A key principle of the Bombay Plan was that the economy could not grow without
government intervention and regulation. Under the assumption that the fledgling Indian industries would
not be able to compete in a free-market economy, the Plan proposed that the future government protect
indigenous industries against foreign competition in local markets.
• The Bombay Plan, seriously took up the question of rapid economic growth and equitable distribution,
even arguing for the necessity of partial nationalization, the public sector, land reform and a series of
workers’ welfare schemes. Hence only option 4 is not correct.
33 www.visionias.in ©Vision IAS

You might also like